Sie sind auf Seite 1von 63

FORMATIVE 2 2014

1. The interview is as much a time for you to learn the details of the job as it is for your potential employer
to find out about you. You will need to obtain specific information about the job itself; including the type
of clients you will be caring for, the people with whom you would be working, the salary and benefits,
and your potential employers expectations of you. Debby, a newly registered nurse is preparing for her very
first job interview. She knows that during this interview, first impressions are made. All of the following are
correct practices during an interview except?
A. Begin with questions about the employers expectations of you.
B. Avoid expressions such as yeah, you know, or like to ease tension.
C. When asking questions to the interviewer, you may asks questions about benefits, day-off or sick time
D. During the interview, use the interviewers last name.
Rationale:
OPTION C- When asking questions to the interviewer, do not begin with questions about vacations,
benefits, or sick time. Beginning with questions about vacations, benefits, or sick time would leave the impression
that these are the most important part of the job to you, not the work itself.
OPTION A- Do not begin with questions about the employers expectations of you. This is a false statement. The
interviewee should begin with questions about the employers expectations of you. This will leave the impression that
you want to know how you can contribute to the organization
OPTION B- Use expressions such as yeah, uh-huh, uh, you know, or like to create a less formal
environment and ease tension. Words like yeah, uh-huh, uh, you know, or like are too casual for an
interview.
OPTION D- During the interview, use the interviewers first name. In an interview, use the interviewers title and
last name as you speak. Never use the interviewers first name unless specifically requested to do so.
The following are a few additional tips about asking questions during a job interview:
Do not begin with questions about vacations, benefits, or sick time. This would leave the impression that
these are the most important part of the job to you, not the work itself.
Do begin with questions about the employers expectations of you. This will leave the impression that you
want to know how you can contribute to the organization.
Be sure you know enough about the position to make a reasonable decision about accepting an offer if one
is made.
Do ask questions about the organization as a whole. The information is useful to you and demonstrates that
you are able to see the big picture.
Do bring a list of important points to discuss to help if you are nervous.
Phrase your questions appropriately and Use appropriate grammar and diction. Words like yeah, uhhuh, uh, you know, or like are too casual for an interview.
During the interview, use the interviewers title and last name as you speak. Never use the interviewers first
name unless specifically requested to do so.
Reference: Essentials of Nursing Leadership and Management by Tappen, R. 2001 by F. A. Davis Company
2. A resume is a form of self-advertisement and is the first impression the recruiter of ones potential
employer will have of you. Through the rsum you are selling yourselfyour skills, talents, and
abilities. AJ, who has just passed the Nursing Licensure Exam is busy preparing his resume and is getting
ready to apply for a job at Cebu Doctors Hospital. He is fully aware that this two-page summary has to work well
enough to get the position he wants. In writing a resume, AJ is right to enact which of the following?
A. Repeat yourself to make your resume more comprehensive.
B. Include personal information such as weight, marital status, and number of children
C. Put the strongest statements/ points at the beginning of your resume
D. Use pompous words to describe your skills and abilities

ANSWER: C
RATIONALE: The question is asking for a POSITIVE statement.
OPTIONS A,B and D are all incorrect statements.
Most professional recruiters and placement services agree on the following tips in preparing a rsum (Anderson,
1992; Rodriquez & Robertson, 1992):
Make sure your rsum is readable. Although most professionals recommend a rsum of no more than two
pages, the length does not appear to be as important as the need for the reader to find the critical information easily.
Is the type large enough for easy reading? Are paragraphs indented or bullets used to set off information, or does the
entire page look like a gray blur? Using bold headings and appropriate spacing can offer relief from lines of gray type,
but be careful not to get so carried away with graphics that your rsum becomes a new art form. The paper should
be an appropriate color such as cream, white, or off-white. Use easily readable fonts and a laser printer. If a good
computer and printer are not available, most printing services prepare rsums at a reasonable cost.
Make sure the important facts are easy to spot. Education, current employment, responsibilities, and facts to
support the experience you have gained from previous positions are important. Put the strongest
statements at the beginning. Avoid excessive use of the word I. If you are a new nursing graduate and have little or
no job experience, list your educational background first. Remember that positions you held before you entered
nursing can frequently support experience that will be relevant in your nursing career. Do let your prospective
employer know how you can be contacted.
Do a spelling and grammar check. Use simple terms, action verbs, and descriptive words. Check your finished
rsum for spelling, style, and grammar errors. If you are not sure how something sounds, get another opinion.
Follow the donts.
Dont include pictures, fancy binders, or personal references.
Dont include salary information or hobbies (unless they have contributed to your work experience).
Dont include personal information such as weight, marital status, and number of children.
Dont repeat yourself just to make the rsum longer. A good rsum is lean and to the point and focuses
on your strengths and accomplishments.
Reference: Essentials of Nursing Leadership and Management by Tappen, R. 2001 by F. A. Davis Company
3. Frederick Herzberg departed from the need theories of motivation and examined experiences that
satisfied or dissatisfied people at work. Known as the Two-factor or Motivational Hygiene Theory, people
have two sets of needs- one related to the avoidance of pain and one related to the desire for
psychological growth. As the Nurse Manager, Nerrie knows that she must make sure that the work is
stimulating and rewarding so that the employees are motivated to work and perform harder and better. She
clearly understands Herzberg theory when she cites which of the following as an example of Motivational
Factors?
A. Fringe benefits
B. Growth and promotional opportunities
C. Interpersonal relations with his peers, superiors and subordinates
D. Company Policies and administrative policies
RATIONALE: the question is asking for an example of Hygiene Factors.
OPTION B- Growth and promotional opportunities
According to Herzberg, the hygiene factors cannot be regarded as motivators. The motivational factors yield
positive satisfaction. These factors are inherent to work. These factors motivate the employees for a superior
performance. These factors are called satisfiers. These are factors involved in performing the job. Employees find
these factors intrinsically rewarding. The motivators symbolized the psychological needs that were perceived as an
additional benefit. Motivational factors include:
Recognition - The employees should be praised and recognized for their accomplishments by the managers.

Sense of achievement - The employees must have a sense of achievement. This depends on the job. There
must be a fruit of some sort in the job.
Growth and promotional opportunities - There must be growth and advancement opportunities in an
organization to motivate the employees to perform well.
Responsibility - The employees must hold themselves responsible for the work. The managers should give
them ownership of the work. They should minimize control but retain accountability.
Meaningfulness of the work - The work itself should be meaningful, interesting and challenging for the
employee to perform and to get motivated.
OPTIONS A, C & D are all Hygiene factors.
Hygiene factors are those job factors which are essential for existence of motivation at workplace. These do
not lead to positive satisfaction for long-term. But if these factors are absent / if these factors are non-existant at
workplace, then they lead to dissatisfaction. In other words, hygiene factors are those factors which when
adequate/reasonable in a job, pacify the employees and do not make them dissatisfied. These factors are extrinsic to
work. Hygiene factors are also called as dissatisfiers or maintenance factors as they are required to avoid
dissatisfaction. These factors describe the job environment/scenario. The hygiene factors symbolized the
physiological needs which the individuals wanted and expected to be fulfilled. Hygiene factors include:
Pay - The pay or salary structure should be appropriate and reasonable. It must be equal and
competitive to those in the same industry in the same domain.
Company Policies and administrative policies - The company policies should not be too rigid.
They should be fair and clear. It should include flexible working hours, dress code, breaks, vacation, etc.
Fringe benefits - The employees should be offered health care plans (mediclaim), benefits for the
family members, employee help programmes, etc.
Physical Working conditions - The working conditions should be safe, clean and hygienic.
The work equipments should be updated and well-maintained.
Status - The employees status within the organization should be familiar and retained.
Interpersonal relations - The relationship of the employees with his peers, superiors and
subordinates should be appropriate and acceptable. There should be no conflict or humiliation
element present.
Job Security - The organization must provide job security to the employees.
Reference: Nursing Management in the New Paradigm, Jones & Bartlett Learning, 1996

4. Burns (2003) viewed leadership as a master discipline that illuminates some of the toughest problems
of human needs and social change. This theory states that the true nature of leadership is not the ability to
motivate people to work hard for their pay but the ability to transform followers to become more self-directed in
all they do.
A. Trait theory
B. Great Man theory
C. Transformational theory
D. Situational Theory
ANSWER: C
RATIONALE: Transformational theory states that the true nature of leadership is not the ability to motivate people to
work hard for their pay but the ability to transform followers to become more self-directed in all they do. Traits theory
states that to become a leader, one must possess certain characteristics or features. Great Man theory believes that
to become a leader one must be great or those belonging to the noble class. The most well-known and used
situational theory involves assessing the nature of the task and the followers motivation or readiness to learn and
using that to determine the particular style the leader should use.

Reference: Guides to Nursing Leadership and Management, Rebecca Jones


5. The total care, or case method was one of the earliest models of nursing care delivery. One nurse
assumes total responsibility for the planning and delivery of care to a particular client or group of clients
and may be used today in community health nursing, in private duty, in intensive care and isolation
units, and in making assignments for students in nursing school. Nurse Nerrie is assigned to a patient who
is in a comatose state in the intensive care unit for the morning shift. She realizes that all of the following is a
disadvantage of this kind of nursing modality aside from?
A. Large number of registered nurses or senior nursing students required for per shift.
B. Consistency of one individual caring for patients in an entire shift
C. During nursing shortages, many hospitals assign health care workers who are not RNs
D. The patient will be nursed by at least 3 nurses during a 24 hour period.
ANSWER: B
RATIONALE: The question is asking for the ADVANTAGE of Case Method Nursing. Options A,C and D are all
disadvantages of this kind of care modality.
The following are the Advantage and Disadvantages of Case Method or Total Care Nursing:
ADVANTAGES
DISADVANTAGES

The organization and work of nurses is


relatively easy for the unit manager,
provided that he has sufficient nurses in the
ward.
The coordination of different services to
provide care for one or two patients is done
by the nurse assigned to them, and thus the
visits of other health team members are
well spaced
It is easy to fix the responsibility for care,
because a specific nurse assumes
accountability for a reasonable length of
time for a specific patient/ patients.
Consistency of one individual caring for
patients in an entire shift. This enables the
patient, family and the nurse to develop a
relationship based on trust.
Assigning patients is simple and direct and
does not require the planning that other
methods of patient care delivery require.

Large number of registered nurses or


senior nursing students required for per
shift. The required number is not usually
available.
The patient will be nursed by at least 3
nurses during a 24 hour period when the 8
hour shift system operates, resulting to
confusion.
During nursing shortages, many hospitals
assign health care workers who are not
RNs to provide most of the nursing care,
thus could result to unsafe care.

Reference: Dimensions of Nursing Management, Juta and Company Ltd, 1998; Leadership Roles and Management
Functions in Nursing: Theory and Application By Bessie L. Marquis, Carol Jorgensen Huston
6. Nonmaleficience is a key ethical principle that guides the nurse to perform actions that would do no
injury or prevent such to their patients. Although it is a simple and straightforward concept, there are
situations derails nurses into violating the principle. An ICU nurse was performing her nursing rounds when
she saw a spill of water beside the bed of a 74-year old confused and agitated patient. Which of the following
situations would incur an act of negligence?
A. the patient had seizures an hour after the rounds and the patient sustained permanent brain damage

B. She forgot to teach proper ROM exercises and use of assistive devices, and the patient sustained a fracture
in the left leg after ambulating.
C. the nurse did not raise the side rails
D. it was her duty to wipe the spilled water but she ignored to do so
Answer: Option B. For an act to be considered negligence, there are 4 elements that are to be present all at the
same time:
ABCD of Malpractice (Negligence)
An injury occurred within the
Breadth of duty of the nurse where
Cause of injury is the failure to perform the
Duty
Choices C and D satisfies only B and D, choice A satisfies only A, while options B satisfies all 4 elements: ABCD
Ethics and Issues in Contemporary Nursing, by Burkhardt and Nathaniel, p.157, 3rd edition, 2008)
7. Every day, nurses encounter situations in which they must make decisions based on the determination
of right and wrong. This desire to maintain clients rights, however, often conflicts with professional
duties and institutional policies. It is essential to balance these two perspectives so that the primary
objective, delivery of quality care, is achieved. You are an out -going nurse from your night duty. While
walking on your way home, you witnessed a motor vehicle crash. You stopped and offered assistance. As a
nurse you know that the following principles applies to your situation excluding?
A. After assessing the situation, the nurse can leave to obtain help.
B. The nurse needs to know the Good Samaritan Act for the state.
C. The nurse is not held liable unless there is gross negligence.
D. The nurse offers to help but cannot insist on helping.
RATIONALE:
OPTION A: Unless there is another equally or more qualified person present, the nurse needs to stay until the injured
person leaves. The nurse should ask someone else to call or go for additional help.
OPTION B: The nurse is subject to the limitation of state law and should be familiar with the Good Samaritan
laws in the specific state. Good Samaritan acts are laws designed to protect health care providers who provide
assistance at the scene of an emergency against claims of malpractice unless it can be shown that there was a gross
departure from the normal standard of care or willful wrongdoing on their part. To encourage citizens to be Good
Samaritans, most states have now enacted legislation releasing a Good Samaritan from legal liability for injuries
caused under such circumstances, even if the injuries resulted from negligence of the person offering emergency aid.
It is important, however, to check your state's statute since some states (e.g., Vermont) require people to stop and
aid persons in danger. It is generally believed that a person who renders help in an emergency, at a level that would
be provided by any reasonably prudent person under similar circumstances, cannot be held liable. The same
reasoning applies to nurses, who are among the people best prepared to help at the scene of an accident. If the level
of care a nurse provides is of the caliber that would have been provided by any other nurse, then the nurse will not be
held liable.
OPTION C: Gross negligence would be described by the individual state law. Gross negligence usually involves
further injury or harm to the person. For example, an automobile may strike an injured child left on the side of the
road when the nurse leaves to obtain help. Most state statutes do not require citizens to render aid to people in
distress. Such assistance is considered more of an ethical than a legal duty
The same client rights apply at the scene of an accident as well as those in the workplace.
.

Guidelines for nurses who choose to render emergency care are as follows:
Limit actions to those normally considered first aid, if possible.
Do not perform actions that you do not know how to do.
Offer assistance, but do not insist. (OPTION D)
Have someone call or go for additional help.
Do not leave the scene until the injured person leaves or another qualified person takes over.
Do not accept any compensation
8. Enormous ethical and legal challenges face nurses today as they strive to provide high-quality care in a
time of shrinking health care budgets. In times like these, nurses need accurate and up-to-date
information on nursing law and ethics. Nurse Debby discovers that the attending physician of his client has
prescribed an unusually large dosage of a medication. Which of the following should Nurse Nerrie do?
A. Read references and administer the appropriate dosage
B. Call the pharmacist who released the medications.
C. Notify the attending physician who prescribed the order.
D. Refuse to administer the medication.
ANSWER: C
RATIONALE: The nurse should call the person who wrote the order for clarification. Administering the medication is
incorrect because knowing the dose is outside the normal range and not questioning the order could lead to client
harm and liability for the nurse. Calling the pharmacist is not the best answer because it will not solve the problem,
and the nurse needs to seek clarification from the person who wrote the order. The nurse should suspend
administration but not refuse to administer the medication until the issue is resolved.
Reference: Kozier 2008, p. 73
9. Your research will dictate the kinds of research methodologies you use to underpin your work and methods you
use in order to collect data. If you wish to collect quantitative data you are probably measuring variables and
verifying existing theories or hypotheses or questioning them. Nurse Prens is assigned to be the leader of
research team organized by the chief nurse of Chong Hua Hospital. They were tasked to initiate a research
paper tackling issues in the operating room complex. Nurse Prens team decided to investigate on the level of
compliance of the operating room nurses to the surgical hand washing protocol of the institution and its relation
to the incidence of post-operative infections among the patients. In the research that the team is working on,
which of the following is the independent variable?
A. Post-operative patients
B. Level of compliance to surgical handwashing protocol
C. Surgical handwashing protocol of the institution
D. Incidence of post-operative infections
ANSWER: B
RATIONALE: The independent variable is the presumed cause (antecedent or influence to the dependent variable)
whereas the dependent variable is the presumed effect. In the case above, the level of compliance to surgical
handwashing protocol of the nurses may or may not have caused post-operative infections to patients.
OPTION D: dependent variable.
VARIABLE is a measurable characteristic that varies. It may change from group to group, person to person,
or even within one person over time.
Independent variables
are those that the researcher has control over. This "control" may involve manipulating existing variables (e.g.,
modifying existing methods of instruction) or introducing new variables (e.g., adopting a totally new method for
some sections of a class) in the research setting. Whatever the case may be, the researcher expects that the

independent variable(s) will have some effect on (or relationship with) the dependent variables.
Dependent variables
show the effect of manipulating or introducing the independent variables. For example, if the independent
variable is the use or non-use of a new language teaching procedure, then the dependent variable might be
students' scores on a test of the content taught using that procedure. In other words, the variation in the
dependent variable depends on the variation in the independent variable.
Reference: ULG, p.435; http://linguistics.byu.edu/faculty/henrichsenl/ResearchMethods/RM_2_14.html
10. Interviews enable face to face discussion with human subjects. If you are going to use interviews you will have to
decide whether you will take notes (distracting), tape the interview (accurate but time consuming) rely on your
memory (foolish) or write in their answers (can lead to closed questioning for times sake). During the analysis of
data gathered, Nurse Prens should be aware of the different types of error that must be avoided in testing the
research hypothesis. Type II error can be described as:
A. null hypothesis is true, and you rejected it.
B. null hypothesis is false, and you accepted it.
C. alternative hypothesis is true, and you rejected it.
D. alternative hypothesis is false, and you accepted it.
ANSWER: B
RATIONALE: OPTION A pertains to type I error.
Errors in hypothesis testing only involve the acceptance or rejection of the null hypothesis.
Type I error, also known as a false positive: the error of rejecting a null hypothesis when it is actually true. In
other words, this is the error of accepting an alternative hypothesis (the real hypothesis of interest) when the
results can be attributed to chance. Plainly speaking, it occurs when we are observing a difference when in truth
there is none (or more specifically - no statistically significant difference).
Type II error, also known as a "false negative": the error of not rejecting a null hypothesis when the alternative
hypothesis is the true state of nature. In other words, this is the error of failing to accept an alternative hypothesis
when you don't have adequate power. Plainly speaking, it occurs when we are failing to observe a difference
when in truth there is one
Reference: Rose Marie Nieswiadomy. Foundations in Nursing Research.p. 305;
http://www.stat.berkeley.edu/users/hhuang/STAT141/Lecture-FDR.pdf
11. In accord with the agencys policies, nurses are required to file incident reports when a situation arises
that could or did cause client harm. Nurse Marcel has just assisted Klaire, her 8 year old client back to bed,
after a fall. She, together with the attending physician assessed Klaire, and determined that the client is not
injured. After completing the incident report, the Nurse Marcel should take which action next?
A. Conduct a staff meeting to describe the fall.
B. Document in the nurse's notes that an incident report was completed.
C. Reassess the client.
D. Contact the nursing supervisor to update information regarding the fall.
ANSWER:C
RATIONALE: The client's fall should be treated as private information and shared on a need to know basis.
Communication regarding the event should involve only those participating in the client's care. An
incident report is a problem-solving document; however, its completion is not documented in the nurse's notes. If the
nursing supervisor has been made aware of the incident, the supervisor will contact the nurse if status update is
desired. After a client's fall, the nurse must frequently reassess the client, because potential complications do not
always appear immediately after the fall.

An incident report (also called an unusual occurrence report) is an agency record of an accident or unusual
occurrence. Incident reports are used to make all facts available to agency personnel, to contribute to statistical data
about accidents or incidents, and to help health personnel prevent future incidents or accidents. All accidents are
usually reported on incident forms. Some agencies also report other incidents, such as the occurrence of client
infection or the loss of personal effects. The nurse completes the following tasks when completing
an incident report:
Identify the client by name, initials, and hospital or identification number.
Give the date. time, and place of the incident.
Describe the facts of the incident. Avoid any conclusions or blame. Describe the incident as you saw it
even if your impressions differ from those of others.
Incorporate the client's account of the incident. State the client's comments by using direct quotes.
Identify all witnesses to the incident.
Identify any equipment by number and any medication by name and dosage.
The report should be completed as soon as possible and filed according to agency policy. Because incident reports
are not part of the client's medical record, the facts of the incident should also be noted in the medical record. Do not
record in the client record that an incident report has been completed because the facts are already documented in
the chart. The purpose of the report form is to alert the risk manager to the event. Incident reports are often reviewed
by an agency risk management committee. which decides whether to investigate the incident further. Nurses may be
required to answer such questions as what they believe precipitated the accident, how it could have been prevented,
and whether any equipment should be adjusted. When an accident occurs, the nurse should first assess the client
and intervene to prevent injury. If a client is injured, nurses must take steps to protect the client, themselves, and
their employer. Most agencies have policies regarding accidents. It is important to follow these policies and not to
assume one is negligent. Although negligence may be involved, accidents can and do happen even when every
precaution has been taken to prevent them.
Reference: Kozier & Erb's Fundamentals of Nursing: Concepts, Process, and Practice by & Shirlee Snyder, Prentice
Hall PTR, 2011
12. Respiratory rate (aka respiration rate, pulmonary ventilation rate or ventilation rate, breathing frequency
(BF) is the number of breaths a living being, such as a human, takes within a certain amount of time
(frequently given in breaths per minute). As a part of the routine physical examination, you are about to take
the vital signs of the patient. However, the client is on her phone talking to her husband. How would you take the
clients respiratory rate?
A. Count it when the client is listening on the phone rather than talking.
B. Tell the client the importance of the assessment and resume it at a later time.
C. Wait at the clients bedside until the phone call is completed and then count the respirations.
D. Record the measurement as deferred since the client is not in respiratory distress because of her
ability to talk to the client and take it later.
ANSWER: D
RATIONALE: Postponing the assessment may seem a controversial answer to some nurses and is definitely a
judgment requiring critical thinking. Unless the client is leaving for the test is immediately and respiration rate is a
critical aspect of the pretest assessment, it is probably not necessary to invade privacy and requires the client to end
the phone cal nor to waste the nurses time waiting at the bedside for the cal to be completed. Because respirations
should be counted with the client at rest, counting during a pause in the conversation doesnt really qualify. Agency
policy would dictate if the deferral should be charted or just the accurate measurement once it has been obtained.
Reference: Fundamentals of Nursing, 7th Edition, Kozier
13. Physical assessment of the thorax and lungs is critical to assessing the clients oxygenation status.
Changes in the respiratory system can come about slowly or quickly. That is why the nurse must be
knowledgeable and skillful in assessing for the normal and abnormal patterns related to oxygenation.
Nurse Vanjie works at the Pulmonary Unit in a Tertiary Specialty Hospital which caters to a variety of patients

with both acute and chronic respiratory diseases. Air rushing through the respiratory tract during inspiration and
expiration generates different breath sounds. Considering that EJ is a healthy male adult, when Nurse Vanjie
auscultates the area above the patients trachea, all but one of the following statements correctly describes the
expected assessment finding?
A. vesicular breath sounds are expected
B. expected sound is high in pitch
C. a loud, harsh sound is expected
D. the auscultated sound is heard longer on expiration than on inspiration
ANSWER: A
RATIONALE: Vesicular breath sounds are heard over the peripheral or bases of the lungs. Bronchial or tubular
breath sounds is expected over the trachea. All other options correctly describe bronchial or tubular breath sounds. It
has a high pitch with a loud, harsh quality. It is heard longer during expiration than inspiration (1:2).
Reference: Fundamentals of Nursing by Kozier et. al. 8th edition, 2008; pp. 469-470, Health Assessment and
Physical Examination by Estes 3rd edition, 2006)
14. Assessment of the thorax includes the inspection of the shape and symmetry of the thorax. Ms. Mian, a
patient with long-standing rickets, is for routine check-up. During the assessment of the thorax, Nurse Vanjie
expects which of the following findings?
A. pectus carinatum
B. barrel chest
C. the ratio of AP-transverse diameter is 1:2
D. None of the above
ANSWER: A
RATIONALE: Pectus carinatum, or pigeon chest, is the marked protrusion of the sternum which increases the AP
diameter of the thorax. Rickets (Vit. D deficiency) results in weak and demineralized bones. This loss of bone
strength allows the intercostal muscles to pull the sternum forward during respiration, resulting in pectus carinatum. A
barrel chest is seen in chronic oxygen insufficiency, as in COPD. Pectus excavatum or funnel chest is often the
result of a congenital anomaly. A ratio of 1:2 in terms of AP-T diameter is normal. (p. 613, Fundamentals of Nursing
by Kozier et. al. 8th edition, 2008; pp. 451-452, Health Assessment and Physical Examination by Estes 3rd edition,
2006)
NORMAL CHEST
BARREL CHEST
PECTUS CARINATUM
PECTUS EXCAVATUM

15. Assessment of the thorax includes the palpation of the tracheal position. Mr. Viton has been recently
admitted to the unit because of chest trauma d/t a vehicular accident. Dyspnea, anxiety, tachypnea and
tachycardia suddenly developed. Hyperresonance of the right lung field is observed. Tension

pneumothorax of the right lung is suspected. Which of the following assessment findings if found by Nurse
Nerrie would indeed confirm the suspicion of tension pneumothorax?
A. tracheal deviation to the right
B. tracheal deviation to the left
C. tracheal deviation is not observed.
D. tracheal position at midline in the suprasternal notch
ANSWER: B
RATIONALE:
Pneumothorax is defined as the presence of air or gas in the pleural cavity (ie, the potential space between
the visceral and parietal pleura of the lung), which can impair oxygenation and/or ventilation. The clinical results are
dependent on the degree of collapse of the lung on the affected side. If the pneumothorax is significant, it can cause
a shift of the mediastinum and compromise hemodynamic stability. Air can enter the intrapleural space through a
communication from the chest wall (i.e. trauma) or through the lung parenchyma across the visceral pleura.
A tension pneumothorax may generate sufficient pressure inside the chest to force the trachea towards the
unaffected side. In this case, the right lung is affected by tension penumothorax and the trachea is expected to
deviate towards the unaffected side, which is the left. Tracheal position at midline in the suprasternal notch is a
normal finding.

Diagnosis: History and physical examination remain the keys to making the diagnosis of pneumothorax.
Examination of patients with this condition may reveal diaphoresis and cyanosis (in the case of tension
pneumothorax).

Findings on lung auscultation vary depending on the extent of the pneumothorax:


Respiratory distress (considered a universal finding) or respiratory arrest
Tachypnea (or bradypnea as a preterminal event)
Asymmetric lung expansion: Mediastinal and tracheal shift to contralateral side (large tension
pneumothorax)
Distant or absent breath sounds: Unilaterally decreased/absent lung sounds common, but decreased air
entry may be absent even in advanced state of pneumothorax
Minimal lung sounds transmitted from unaffected hemithorax with auscultation at midaxillary line
Hyperresonance on percussion: Rare finding; may be absent even in an advanced state
Decreased tactile fremitus
Adventitious lung sounds: Ipsilateral crackles, wheezes
Cardiovascular findings may include the following:
Tachycardia: Most common finding; if heart rate is faster than 135 beats/min, tension pneumothorax likely
Pulsus paradoxus
Hypotension: Inconsistently present finding; although typically considered a key sign of tension
pneumothorax, hypotension can be delayed until its appearance immediately precedes cardiovascular
collapse
Jugular venous distention: Generally seen in tension pneumothorax; may be absent if hypotension is severe
Dx Study:
Chest radiography: Anteroposterior and/or lateral decubitus films
Contrast-enhanced esophagography: If emesis/retching is the precipitating event
Chest computed tomography scanning: Most reliable imaging study for diagnosis of pneumothorax but not
recommended for routine use in pneumothorax
Chest ultrasonography

Management: Although there is general agreement on the management of pneumothorax, a full consensus
about management of initial or recurrent pneumothorax does not exist. Rather, many clinicians use a risk
stratification framework as well as other approaches for choosing among options to restore lung volume and an
air-free pleural space and to prevent recurrences.[54]The range of medical therapeutic options for pneumothorax
includes the following:

Watchful waiting, with or without supplemental oxygen


Simple aspiration
Tube drainage, with or without medical pleurodesis
Surgery
If the patient has had repeated episodes of pneumothorax or if the lung remains unexpanded after 5 days with a
chest tube in place, operative therapy such as the following may be necessary:
Thoracoscopy: Video-assisted thoracoscopic surgery (VATS)
Electrocautery: Pleurodesis or sclerotherapy
Laser treatment
Resection of blebs or pleura
Open thoracotomy
Reference: (p. 465, Health Assessment and Physical Examination by Estes 3rd edition, 2006)
http://emedicine.medscape.com/article/424547-overview

16. Atelectasis is defined as the lack of gas exchange within alveoli, due to alveolar collapse or fluid consolidation.
Teje, a 4-year old child, was rushed to the ER after swallowing several pieces of clay. After undergoing x-ray, the
child is diagnosed with atelectasis of the right lung d/t to complete obstruction in the right bronchus. Stat surgery
is to be done. Which of the following assessment findings would Nurse Vits most likely find?
A. tracheal shift to the left, breath sounds absent in both lung fields, dull percussion in the right lung
B. tracheal shift to the left, breath sounds absent in the left lung, present in the left, hyperresonance percussion
in the left lung
C. tracheal shift to the right, breath sounds absent in the right lung, present in the left, resonance percussion in
the left lung
D. tracheal shift to the right, breath sounds absent in the right lung, present in the left, dull percussion on the
right lung
ANSWER: D
RATIONALE: In atelectasis, fewer aerating alveoli are present and therefore have a lower pressure. Therefore, the
trachea is slightly pushed by the healthy lung to the affected side, which contains less air and lesser pressure. Breath

sounds are absent in the right lung because alveoli are not ventilated because air does not reach them. Breath
sounds are present in the left lung because the alveoli are still ventilated as the bronchus is unobstructed. Dull
percussion is elicited because the alveoli are collapsed and air is not present in the affected lung.
Atelectasis is the collapse of part or (much less commonly) all of a lung. Is caused by blockage of the air
passages (bronchus or bronchioles) or by pressure on the outside of the lung. It is common after surgery, or in
patients who were in the hospital.
Risk factors: Anesthesia; Foreign object in the airway (most common in children); Lung diseases; Mucus that
plugs the airway; Pressure on the lung caused by a buildup of fluid between the ribs and the lungs (called
a pleural effusion); Prolonged bed rest with few changes in position; Shallow breathing (may be caused by
painful breathing); Tumors that block an airway
Symptoms: Breathing difficulty, Chest pain, Cough, Fever, low-grade, usually after surgery

Mgt: The goal of treatment is to re-expand the collapsed lung tissue. If fluid is putting pressure on the lung,
removing the fluid may allow the lung to expand. The following are treatments for atelectasis:
Clap (percussion) on the chest to loosen mucus plugs in the airway
Perform deep breathing exercises (with the help of incentive spirometry devices)
Remove or relieve any blockage in the airways by bronchoscopy or another procedure.
Tilt the person so the head is lower than the chest (called postural drainage). This allows mucus to drain
more easily.
Treat a tumor or other condition, if there is one
Turn the person to lie on the healthy side, allowing the collapsed area of lung to re-expand
Use aerosolized respiratory treatments (inhaled medications) to open the airway
Use other devices that help increase positive pressure in the airways and clear fluids (positive expiratory
pressure [PEP] devices
Reference: (p. 473, Health Assessment and Physical Examination by Estes 3rd edition, 2006)
http://www.nlm.nih.gov/medlineplus/ency/article/000065.htm
17. Mammography is a radiographic technique used to help detect breast cysts or tumors, especially those not
palpable on physical examination. Nurse Ian is instructing a patient about the mammogram. Which of the
following instruction should the nurse include during patient teaching?
A. Do not drink fluids for four hours prior to the test
B. The X-ray procedure takes two hours to finish.
C. No pain or discomfort will be experienced during the procedure
D. Do not use deodorant in the underarm area before the test
ANSWER: D
RATIONALE:
A mammogram is an X-ray image of your breast used to screen for breast cancer. Mammograms play a key role in
early breast cancer detection and help decrease breast cancer deaths. Mammography can be used either for
screening or for diagnostic purposes in evaluating a breast lump:
1. Screening mammography. Screening mammography is used to detect breast changes in women who
have no signs or symptoms or observable breast abnormalities. The goal is to detect cancer before clinical
signs are noticeable.
2. Diagnostic mammography. Diagnostic mammography is used to investigate suspicious breast changes,
such as a breast lump, breast pain, an unusual skin appearance, nipple thickening or nipple discharge. It's
also used to evaluate abnormal findings on a screening mammogram. A diagnostic mammogram includes
additional mammogram images.
Points to remember:

One breast at a time is rested on a flat surface that contains the x-ray plate.
Do not use deodorant, perfume, powders, or ointments under your arms or on your breasts on the day of
the mammogram. Metallic particles in powders and deodorants could be visible on your mammogram and
cause confusion. (OPTION D)
Remove all jewelry from your neck and chest area.
Tell your health care provider and the radiologist if you are pregnant or breastfeeding.
Note: The level of radiation is low, and any risk from mammography is very low. If you are pregnant and need to
have an abnormality checked, your belly area will be covered and protected by a lead apron. Routine screening
mammography is not done during pregnancy or while breastfeeding.
The metal may feel cold. When the breast is pressed down, you may have some pain. (OPTION C) Taking an
over-the-counter pain medication, such as aspirin, acetaminophen (Tylenol, others) or ibuprofen (Advil, Motrin
IB, others), about an hour before your mammogram might ease the discomfort of the test.
Mammography is recommended for:
1. Women starting at age 40, repeated every 1 - 2 years. (This is not recommended by all expert
organizations.)
2. All women starting at age 50, repeated every 1 - 2 years.
3. Women with a mother or sister who had breast cancer at a younger age should consider yearly
mammograms. They should begin earlier than the age at which their youngest family member was
diagnosed.
The entire procedure usually takes less than 30 minutes. If the views are inadequate for technical reasons, you
may have to repeat part of the test (OPTION B).
Reference:
Lippincott
Manual
of
Nursing
Practice
8th
edition
pp
857;
http://www.nlm.nih.gov/medlineplus/ency/article/003380.htm
18. Collecting objective data is essential for a complete nursing assessment. The nurse must have knowledge of
and skill in three basic areas to become proficient in collecting objective data: necessary equipment and how to
use it; preparing the setting, oneself, and the client for the examination; and how to perform the four basic
assessment techniques. Nurse Bhum is caring for a client who has suffered a severe cerebrovascular accident.
During routine assessment, the nurse notices Cheyne- Strokes respirations. Cheyne-strokes respirations are:
A. Rapid, deep breathing with abrupt pauses between each breath.
B. Rapid, deep breathing and irregular breathing without pauses.
C. Shallow breathing with an increased respiratory rate.
D. A progressively deeper breaths followed by shallower breaths with apneic periods.
ANSWER: A
RATIONALE: A progressively deeper breaths followed by shallower respirations with apnea periods. CheyneStokes respiration is characterized by alternating episodes of apnea (cessation of breathing) and periods of deep
breathing. Deep respirations become increasingly shallow, followed by apnea that may last approximately 20
seconds. The cycle repeats after each apneic period. The duration of the period of apnea may vary and may
progressively lengthen; therefore, it is timed and reported. Cheyne-Stokes respiration is usually associated with heart
failure and damage to the respiratory center (drug-induced, tumor, trauma)
OPTION A- Biots respirations are rapid, deep breathing with abrupt pauses between each breath, and equal depth
between each breath. Biots respirations, or cluster breathing, are cycles of breaths that vary in depth and have
varying periods of apnea. Biots respirations are seen with some central nervous system disorders.
OPTION B- Kussmauls respirations are rapid, deep breathing without pauses. Hyperventilation that is marked by an
increase in rate and depth, associated with severe acidosis of diabetic or renal origin, is called
Kussmauls respiration.
OPTION C- Tachypnea is shallow breathing with increased respiratory rate.
Reference: Brunner and Suddarth's Textbook of Medical-Surgical Nursing by Suzanne C. O'Connell Smeltzer,
Brenda G. Bare, Janice L. Hinkle, Ph.D., Kerry H. Cheever, Ph.D., Lippincott Williams & Wilkins, 2010

19. The heart is a hollow, muscular organ located in the center of the thorax, where it occupies the space
between the lungs (mediastinum) and rests on the diaphragm. It weighs approximately 300 g (10.6 oz),
although heart weight and size are influenced by age, gender, body weight, extent of physical exercise
and conditioning, and heart disease. The heart pumps blood to the tissues, supplying them with oxygen
and other nutrients. During the physical examination of the well adult client, the health care provider
auscultates the heart. When the stethoscope is placed on the 2nd LICS left Intercostal Space, Left Sternal
Border, which valve closure is best evaluated?
A. Tricuspid
B. Pulmonic
C. Aortic
D. Mitral
RATIONALE:
OPTION B- Pulmonic closure is best heard at the 2nd LICS left Intercostal Space, Left Sternal Border.
OPTION A- The sound created by the closure of the tricuspid valve is heard at the 5th left ICS at the Left Sternal
Border.
OPTION C- Aortic closure is best herad at the 2nd Right Intercostal Space, Right Sternal Border.
OPTION D- Mitral Valve is best heard at the PMI landmark (apex)
Heart sounds are produced by valve closure. The opening of valves is silent. Normal heart sounds,
characterized as lub dubb (S1 and S2), and, occasionally, extra heart sounds and murmurs can be auscultated with
a stethoscope over the precordium, the area of the anterior chest overlying the heart and great vessels.
The first heart sound (S1) is the result of closure of theAVvalves: the mitral and tricuspid valves. S1
correlates with the beginning of systole. S1 (lub) is usually heard as one sound but may be heard as two sounds. If
heard as two sounds, the first component represents mitral valve closure (M1), and the second component
represents tricuspid closure (T1).M1 occurs first because of increased pressure on the left side of the heart and
because of the route of myocardial depolarization. S1 may be heard over the entire precordium
but is heard best at the apex (left MCL, fifth ICS).
The second heart sound (S2) results from closure of the semilunar valves (aortic and pulmonic) and
correlates with the beginning of diastole. S2 (dubb) is also usually heard as one sound but may be heard as two
sounds. If S2 is heard as two sounds, the first component represents aortic valve closure (A2) and the second
component represents pulmonic valve closure (P2). A2 occurs first because of increased pressure on the left
side of the heart and because of the route of myocardial depolarization. If S2 is heard as two distinct sounds, it is
called a split S2. A splitting of S2 may be exaggerated during inspiration and disappear during expiration. S2 is heard
best at the base of the heart.
S3 and S4 are referred to as diastolic filling sounds or extra heart sounds, which result from ventricular
vibration secondary to rapid ventricular filling. If present, S3 can be heard early in diastole, after S2 . S4 also results
from ventricular vibration but, contrary to S3, the vibration is secondary to ventricular resistance (noncompliance)
during atrial contraction. If present, S4 can be heard late in diastole, just before S1. S3 is often termed ventricular
gallop, and S4 is called atrial gallop.
REFERENCE: Brunner and Suddarth's Textbook of Medical-Surgical Nursing by Suzanne C. O'Connell Smeltzer,
Brenda G. Bare, Janice L. Hinkle, Ph.D., Kerry H. Cheever, Ph.D., Lippincott Williams & Wilkins, 2010
20. Whenever possible, the nurse implements strategies to prevent infection. If infection cannot be
prevented, the nurse's goal is to prevent the spread of the infection within and between persons and to
treat the existing infection. After teaching a client and family strategies to prevent infection prevention, which
statement by the client would indicate effective learning has occurred?
A. "We must wash or peel all raw fruits and vegetables before eating."
B. "We will use antimicrobial soap and hot water to wash our hands at least three times per day."
C. "A wound or sore is not infected unless we see it draining pus."

D. "We should not share toothbrushes but it is fine to share towels and washcloths."
ANSWER: A
RATIONALE: Raw foods touched by human hands can carry significant infectious organisms and must be washed or
peeled.
OPTION B- Antimicrobial soap is not indicated for regular use and may lead to resistant organisms. Hand hygiene
should occur as needed. Hot water can dry and harm skin, increasing the risk of infection
OPTION C- Clients should learn all the signs of inflammation and infection (e.g., redness, swelling, pain, heat) and
not rely on the presence of pus to indicate this.
OPTION D- People should not share washcloths or towels
STANDARD PRECAUTIONS are designed for all clients in hospital and applies to (a) blood; (b) all body fluids,
excretions, and secretions except sweat; (c) nonintact (broken) skin; and (d) mucous membranes. Designed to
reduce risk of transmission of microorganisms from recognized and unrecognized sources.
Perform proper hand hygiene after contact with blood, body fluids, secretions, excretions, and contaminated
objects whether or not gloves are worn.
o Perform proper hand hygiene immediately after removing gloves.
o Use a nonantimicrobial product for routine hand hygiene.
o Use an antimicrobial agent or an antiseptic agent for the control of specific outbreaks of infection.
Wear clean gloves when touching blood, body fluids, secretions, excretions, and contaminated items (i.e.,
soiled gowns).
o Clean gloves can be unsterile unless their use is intended to prevent the entrance of
microorganisms into the body.
See the discussion of sterile gloves in this chapter.
o Remove gloves before touching noncontaminated items and surfaces.
o Perform proper hand hygiene immediately after removing gloves.
Wear a mask, eye protection, or a face shield if splashes or sprays of blood, body fluids, secretions, or
excretions can be expected.
Wear a clean, nonsterile, water-resistant gown if client care is likely to result in splashes or sprays of blood,
body fluids, secretions, or excretions. The gown is intended to protect clothing.
o Remove a soiled gown carefully to avoid the transfer of microorganisms to others {i.e., clients or
other health care workers).
o Cleanse hands after removing gown.
Handle client care equipment that is soiled with blood, body fluids, secretions, or excretions carefully to
prevent the transfer of microorganisms to others and to the environment.
o Make sure reusable equipment is cleaned and reprocessed correctly.
o Dispose of single-use equipment correctly
Handle all soiled linen as little as possible. Do not shake it. Bundle it up wi th the clean side out and dirty
side in, and hold away from self so that the nurse's uniform or clothing is not contaminated.
Place used needles and other "sharps" directly into puncture-resistant containers as soon as their use is
completed. Do not attempt to recap needles or place sharps back in their sheaths using two hands; use the
one handed scoop technique or other safety device. Using two hands can result in a needle stick puncture
injury if the nurse accidentally misses the cover.
Reference: Kozier & Erb's Fundamentals of Nursing: Concepts, Process, and Practice by Audrey Berman, Ph.D.
RN, Shirlee Snyder, Prentice Hall PTR, 2011 ;
21. Urinary tract infections (UTI) are the most commonly reported bacterial infection. E coli causes 90% of them,
some of which are hospital acquired which are more difficult to treat. Other UTI are caused by Pseudomonas
aeruginosa, Serratia, and Enterobacter. A 29 year old client presented to the Emergency Department with
painful urination for two days. She didnt have any other associated symptoms. She appeared in no distress and
had no abdominal or flank tenderness on examination. The clean catch urine result showed greater than 25,000

white blood cells with few red blood cells and epithelial cells. A pregnancy test was done which resulted
negative. The physician has ordered Pyridium (phenazopyridine). As a nurse you are knowledgeable enough
about the medication when you exclude which information?
A. The urine will have a redorange color.
B. Driving or perform any activity that requires mental alertness.
C. It is allowed to use contact lenses while using this medication.
D. Tell your doctor or pharmacist if you are allergic to it or if you have any other allergies.
ANSWER: C
RATIONALE: Pyridium (phenazopyridine) is used frequently for UTI but does not itself have any antibacterial
activity. It is excreted in the urine where it exerts a topical analgesic effect. It is indicated for symptomatic relief of
pain, burning, frequency and urgency caused by the irritation that the infection produces in the urinary tract mucosa.
Phenazopyridine is an azo dye. It can dye your urine and tears orange-red. This may stain clothing and
contact lenses. Do not wear contact lenses while using this medication. Urine and tears will return to normal color
after the medication is stopped (OPTIONS A AND C).
This drug may make you dizzy. Do not drive, use machinery, or do any activity that requires alertness until
you are sure you can perform such activities safely (OPTION B)
Before taking phenazopyridine, tell your doctor or pharmacist if you are allergic to it; or if you have any other
allergies. This product may contain inactive ingredients, which can cause allergic reactions or other problems
(OPTION D)
Reference: Drug Therapy in Nursing By Diane S. Aschenbrenner, Samantha J. Venable Lippincott Williams &
Wilkins, 2009 ; http://www.rxlist.com/pyridium-drug/consumer-side-effects-precautions.htm
22. Absorption begins where the drug is administered. The absorption rate of a drug is influenced by a number of
factors that might increase or decrease the rate, This is similar to how more gasoline is used to drive at faster
speeds. Absorption is affected by many factors that include pain, stress, hunger, fasting, food, and pH. The
physician has prescribed Gantrisin (sulfasoxazole) 1g in divided doses for a client with a urinary tract infection.
The nurse does not need further teaching when she states that the drug must be administered when?
A. with meals or a snack
B. 30 minutes before meals
C. 30 minutes after meals
D. at bedtime
RATIONALE:
OPTION B- Gantrisin and other sulfa drugs should be given 30 minutes before meals, to enhance absorption. Other
instructions that needs to be taught to the client includes:
Take the drug with 8 ounces of fluid/ water on an empty stomach.
Follow the physicians prescribed duration of regimen.
Stop the drug if rashes start to appear (AR: allergic reaction)
Stay out of the sun and wear sunscreen and protective clothing if you have to be exposed (AR:
photosensitivity)
Drink 8-10 glasses of water per day (AR: crystalluria)
Reference: Pharmacology by Marilyn J. Herbert-Ashton, Nancy Elaine Clarkson, Jones & Bartlett Learning, 2007;
http://www.drugs.com/pro/gantrisin.html
23. Cardiac catheterization involves passing a thin flexible tube (catheter) into the right or left side of the heart,
usually from the groin or the arm. The client is scheduled for a cardiac catheterization to diagnose the extent of
coronary artery disease. Nurse JM tells the client to report immediately the following sensation when felt during
the procedure?
A. Pressure at the insertion site

B. Warm feeling
C. Flushed feeling
D. Chest pain
ANSWER: D
RATIONALE: The client is taught to immediately report chest pain or any unusual sensations. The client is informed
that a warm, flushed feeling may accompany dye injection and is normal. The client also is told that he or she may be
asked to cough or breathe deeply from time to time during the procedure. Because a local anesthetic is used, the
client is expected to feel pressure at the insertion site.
The test may last 30 - 60 minutes. If you also need special procedures, the test may take longer. If the
catheter is placed in your groin, you will usually be asked to lie flat on your back for a few hours after the test to avoid
bleeding.
Notes:
Preparation before the test:
1. NPO 6-8 hours
2. Tell your doctor if you:
b. Are allergic to seafood or any medications
c. Have had a bad reaction to contrast dye or iodine in the past
d. Take any medicines, including Viagra or other drugs for erectile dysfunction
e. Might be pregnant

Reference:http://books.google.com.ph/books?id=Kgdh_dV5odoC&printsec=frontcover&dq=cardiac+catheterization&
hl=en&sa=X&ei=kfnYUuLiJMjIkQWjyYCABA&ved=0CDEQ6AEwAQ#v=onepage&q=cardiac%20catheterization&f=fal
se
24. The physician ordered Neomycin, an aminoglycoside antibiotic, to Patient Pandaren. You know this is indicated
for patients with liver cirrhosis because it:
A. Prevents formation of bile
B. Prevents formation of urea
C. Prevents formation of ammonia
D. Prevents sepsis
ANSWER: C
RATIONALE: Neomycin destroys intestinal flora, which breaks down protein and in the process gives off ammonia.
Ammonia at this time is poorly detoxified by the liver and can build up to toxic levels.
Reference: Donna D. Ignatavicius& M. Linda Workman: Medical-Surgical Nursing: Patient-Centered Collaborative
Care, 6th Edition pp. 137
25. You have a patient who is taking 0.25 mg tablets of Digoxin. Which of the following events will alert you to
immediately monitor for digoxin toxicity?
A. Seizure episode 2 hours ago
B. Severe diarrhea
C. Initiation of spironolactone therapy
D. Recent diagnosis of acute renal failure

ANSWER: B
RATIONALE: Digoxin toxicity is often precipitated by hypokalemia. Option B is a cause of hypokalemia. All the other
options are events that may lead to hyperkalemia. In addition, one of the most common immediate manifestations of
digoxin toxicity involves the GIT.
Reference: Clayton 2007, p. 458; Porth 2005, p. 769-772
26. The goals, principles and strategies of PHC are not entirely new to nursing. They are parallel and are basically
akin to those of CHN. The provision of PHC is a natural extension of nursing practice, especially as it applies to
community health. Nursing practice in PHC is basically the practice of CHN. Nurse Landa needs no further
teaching when she says that the goal of Primary Health Care (PHC) is?
A. Health for All Filipinos and Health in the Hands of the People by the year 2020
B. Health for All Filipinos
C. To strengthen the health care system by increasing opportunities and supporting conditions wherein people
will manage their own health care
D. Ensure accessibility and quality of health care to improve the quality of life of all Filipinos, especially the
poor.
ANSWER: A
RATIONALE: This is the GOAL of Primary Health Care.
OPTION B- Health for All Filipinos is the VISION of the Department of Health
OPTION C- The mission of PHC is to strengthen the health care system by increasing opportunities and supporting
conditions wherein people will manage their own health care.
OPTION D- This is the MISSION of the Department of Health
Reference: Community Health Nursing Services in the Department of Health Philippines, 9th Edition, 2000
27. The ultimate goal of primary health care is better health for all. WHO has identified five key elements to
achieving that goal, one of which is reducing exclusion and social disparities in health (universal coverage
reforms) All of the following concepts are included in PHC, EXCEPT:

A.
B.
C.
D.

Community Organizing is at center of this strategy


Provision of the basic needs of the community is the key approach
Peoples participation is a cornerstone of this strategy
Use of appropriate technology is one of the tenets of this approach

ANSWER: B
Rationale: Goal of PHC: Health for ALL Filipinos and Health in the Hands of the People by the Year 2020
Mission of PHC: To strengthen the health care system by increasing opportunities and supporting the
conditions wherein people will manage their own health care.
Concept of PHC: partnership and empowerment of the people being the core strategy in the provision of
essential health care services
Framework of meeting the goal of PHC: Organizational Strategy
Option B- Provision of needs is not the key approach instead, PHC aims to put the responsibility for health
on the individual, his family, and the community. It includes full participation and active involvement of the
community towards the development of self-reliant people, capable of achieving an acceptable level of
health and well being.

28. Primary Health Care was declared during the First International Conference on Primary Health Care held in
Alma Ata, USSR on September 6-12, 1978 by WHO. The concept of PHC is characterized by partnership and
empowerment of the people that shall permeate as the core strategy in the effective provision of essential health
services that are community based, accessible, acceptable and sustainable at a cost which the community and
the government can afford. Which of the following scenarios is not included in the components or elements of
PHC?
A. Nurse Margo who makes sure that there is a correct distribution of anti-tuberculosis medications for all
patients positive of TB.
B. Nurse Edith, who gives CPR to a community folk who suddenly collapsed, is not breathing and with
diminishing pulse.
C. Nurse Gru, who conducts health teaching regarding the importance of immunization for children and the
diseases that would be prevented if immunization is completed.
D. Nurse Agnes who starts a daily exercise program in order to decrease the probability of developing
myocardial infarction in his community.
ANSWER: D
RATIONALE: Distribution of anti-TB drugs is under the element provision of essential drugs. Giving CPR also
means giving emergency treatment to the members of the community and is an element of PHC. Health education,
immunization and child health are also included in the components of PHC. Control or prevention of
noncommunicable/ lifestyle diseases (like cardiovascular disorders) is not part of the elements of PHC.
Elements/components of Primary Health Care:
1. Environmental sanitation (adequate supply of safe water and good waste disposal)
2. Control of communicable diseases
3. Immunization
4. Health education
5. Maternal and child health and Family planning
6. Adequate food and Proper nutrition
7. Provision of medical care and Emergency Treatment
8. Treatment of Locally endemic diseases
9. Provision of essential drugs
Reference: Public Health Nursing in the Philippines, by Jean Reyala
29. The potential role and contribution of nursing in PHC is well-recognized and acknowledged. Nurses, by sheer
force of their number, their closeness to prevailing health and related problems and the intimate nature of the
services they provide to individual clients/patients and families could be, in the words of the former Director
General of WHO, Dr. Halfdan Mahler a powerhouse for change. Among the responsibilities of nursing practice
in PHC are the training, support and supervision of auxiliary nursing personnel and other community based
health workers. Who among the following is excluded among the two levels of Primary Health Care Workers?
A. Vito Cruz, a Rural sanitary Inspector
B. Aling Chaka, a Traditional Birth Attendant/Hilot
C. Vitz, a Midwife
D. Mendiola, a newly Registered Pharmacist
ANSWER: C
RATIONALE: OPTION C- A pharmacist is not included in the intermediate level health workers .
In general, there are two levels of Primary Health Care workers;

1. Village or Barangay Health Workers (V/BHWs0- this refers to trained community health workers or health
auxiliary volunteer or a traditional birth attendant or healer.
2. Intermediate Level health workers- General Medical Practitioners or their assistants, Public Health Nurse, Rural
sanitary Inspectors and Midwives may compose these groups.
Reference: Public Health Nursing in the Philippines, by Jean Reyala
30. Immunization is the process by which vaccines are introduced into body before infection sets in, to induce
immunity thereby causing the recipients immune system to produce antibodies. Newborns, infants and pregnant
women are of highest priority because of their vulnerability to diseases. Nurse Noval is a Rural Health Nurse
assigned to Barangay Espanya. It is vaccination day today at the RHU and a number of patients have already
lined up. The RHU is offering vaccinations for both infants and pregnant women. Mrs. Tolentina has brought to
the RHU her 2-month old son. Upon interview, Nurse Noval learns that the child has not yet received any
vaccine since he was born. She said, Totoo po ba nars ang sinabi ng kapitbahay ko na DPT at OPV na lang
daw ang pwedeng ibigay sa anak ko? Di pa po sya natuturukan ng BCG at Hep B eh. The most appropriate
response of the nurse would be:
A. Sana po pinabakunahan nyo sya na BCG at Hep B agad agad. DPT at OPV na lang po ang maibibigay ko
ngayon.
B. Hep B na lang po ang pwede ibigay ang BCG ay ibibigay na lang kapag sya nasa elementary na
C. Maaari po nating ibigay ang Hepa-B at BCG na bakuna kasabay ng DPT at OPV ngayong araw na ito.
D. Dapat po ay magkahiwalay ang bakuna ng DPT at OPV sa Hep B at BCG. Uunahin na lang po natin ang
Hep B at BCG, babalik na lang po kayo sa susunod na lingo para sa DPT at OPV
ANSWER: C
RATIONALE: The minimum ages of the first dose of vaccines are the following: BCG is birth or anytime after birth,
Hepa B is at birth, and DPT and OPV are both six weeks. If the child is not able to receive the vaccines within the
minimum age, they can be given at a later age, as with the child in the situation. All 4 vaccines can be started at 2
months. (p. 149, Public Health Nursing in the Phillpines by Cuevas, et.al., 10th edition, 2007)
31. Mrs. Lacson, a 44-year old pregnant woman (G6 P5) is about to receive her fourth dose of Tetanus Toxoid
Vaccine. During the interview and assessment before the vaccine is given, which of the following statements
made by the patient would indicate additional health education regarding the said vaccine is needed?
A. Good thing that I remembered to come back here in the RHU, since I got my last shot of Tetanus vaccine 6
months ago.
B. I am happy that my sixth child would be safe from tetanus.
C. I have peace of mind because when I receive this 4th vaccine, I need not worry about contracting tetanus
until I reach 54 years old.
D. I presently have 95% protection against tetanus.
Answer: A
RATIONALE: When receiving TT4, the minimum interval should be at least 1 year after the patient has received TT3.
Since the patient has not yet received TT4, she has 95% protection against tetanus, and the child that will be born
will be protected to neonatal tetanus as well. When the patient receives TT4, she will receive protection against
tetanus for 10 years. (p. 150, Public Health Nursing in the Phillpines by Cuevas, et.al., 10th edition, 2007)
32. Health problems that are beyond the capability of PHC units and beyond the competence of PHC workers are
referred to a intermediate health facility, usually a Rural Health Unit (RHU) located in a town or poblacion. There

are also different levels of health care in the referral system of the Philippine Health Care Delivery System. Who
among the following persons gives secondary level of care to his/her patients:
A. Nurse Yham who performs the IMCI strategy to her pediatric patients in the clinic.
B. Dr. Lloyd who performs emergency wound debridement to his patient in the RHU.
C. Clinical nurse specialist Nancy who works at the National Kidney Institute and takes care of patients under
hemodialysis.
D. Dr. Bong who performs herniorrhaphy to his patient referred from an RHU.
ANSWER: D
RATIONALE:
Levels of Health Care and Referral System:
1. Primary level of care
Primary care is devolved to the cities and the municipalities. It is health care provided by center physicians
(Dr. Llyod), public health nurses (Nurse Yham), rural health midwives, barangay health workers, traditional
healers and others at the barangay health stations and rural health units. The primary health facility is
usually the first contact between the community members and the other levels of health facility.
2. Secondary level of care
Secondary care is given by physicians with basic health training. This is usually given in health facilities
either privately owned or government operated such as infirmaries, municipal and district hospitals and outpatient departments of provincial hospitals. This serves as a referral center for the primary health facilities
(Dr. Bhong). Secondary facilities are capable of performing minor surgeries and perform some simple
laboratory examinations.
3. Tertiary level of care
Tertiary care is rendered by specialists (Nurse Specialist Nancy) in health facilities including medical centers
as well as regional and provincial hospitals, and specialized hospitals such as the Philippine Health Center.
The tertiary health facility is the referral center for the secondary care facilities. Complicated cases and
intensive care requires tertiary care and all these can be provided by the tertiary care facility.
Reference: Public Health Nursing in the Philippines, by Jean Reyala
33. The Philippines is tasked to reduce the maternal mortality ratio (MMR) by three quarters by 2015 to achieve its
Millennium Development Goal. The overall goal of the Maternal Health Program is to improve the survival, health
and well-being of mothers and unborn through a package of services for the pre-pregnancy , prenatal, natal and
post natal stages. There are other conditions that might occur among pregnant women. These conditions may
endanger her health and complications could occur. All of the following interventions are inappropriate, except
for one:
A. Oral rehydration solution should be given immediately to a woman who is unconscious or has convulsions.
B. Ergometrine is given to all pregnant women who experiences postpartum bleeding.
C. Mebendazole is given to all pregnant women who complain of intestinal parasite infections.
D. None of the above
ANSWER: D
RATIONALE: The question is looking for a positive answer. Oral rehydration solutions are never given to
unconscious patients or patients with convulsions because it may lead to aspiration. Ergometrine is given to pregnant
women who experiences postpartum bleeding, but it is contraindicated to patients with eclampsia, preeclampsia or
hypertension because it would further aggravate the conditions. Mebendazole is not given to all pregnant women
because mebendazole is not given during the first trimester of pregnancy. It may cause congenital problems in the
baby.

34. As part of primary health care and because of the increasing cost of drugs, the use of locally available medicinal
plants has been advocated by the Department of Health. Which of the following is not true regarding the use of
herbal medicines advocated by the DOH?
A. Tsaang-gubat as a diuretic.
B. Lagundi as treatment for asthma, cough, and fever.
C. Yerba Buena as analgesic in different parts of the body.
D. Niyug-niyogan as an anti-helmentic.
ANSWER: A
RATIONALE:
10 Herbal Medicines Approved by DOH:
S ambong (Blumea Balsamifera): English name: "Ngai camphor or Blumea camphor" is a Philippine herbal medicine
used to treat kidney stones, wounds and cuts, rheumatism, anti-diarrhea, anti spasms, colds and coughs
and hypertension; anti-edema, diuretic.
A mpalaya (Momordicacharantia): Common names include "bitter melon " or "bitter gourd " in English. This
Philippine herbal medicine has been found to be effective in the treatment of diabetes (diabetes mellitus),
hemofrhoids, coughs, burns and scalds, and being studied for anti-cancer properties.
N iyug niyugan (Quisqualisindica L.): is a vine known as "Chinese honey suckle". This Philippine herbal medicine is
used to eliminate intestinal parasites.
T saang Gubat (Ehretiamicrophylla Lam.): English :"Wild tea" is a Philippine herbal medicine taken as tea to treat
skin allergies including eczema, scabies and itchiness wounds in child birth
A kapulko (Cassia Alata): a medicinal plant called "ringworm bush or schrub" and "acapulco" in English, this Philippine
herbal medicine is used to treat tinea infections, insect bites, ringworms, eczema, scabies and itchiness.
L agundi (Vitex negundo): known as "5-leaved chaste tree" in english is used in Philippine herbal medicine to
treat cough, colds and fever. It is also used as a relief for asthma & pharyngitis, rheumatism, dyspepsia,
boils, and diarrhea.
U lasimang Bato (Peperomia pellucida): is a Phillipine herbal medicine known for its effectivity in treating arthritis
and gout.
B ayabas (Psidium guajava): guava in English. It is primarilyused as an antiseptic, to disinfectwounds. Also, it can
be used as amouth wash to treat tooth decayand gum infection.
B awang (Allium sativum): Common name in english is "Garlic". Bawang is a used in Philippine herbal medicine to
treat infection with antibacterial, antiinflammatory, anti-cancer and anti-hypertensive properties. It is widely
used to reduce cholesterol level in blood.
Y erba Buena (Clinopodiumdouglasii): commonly known as Peppermint, is used in Philippine herbal medicine as
analgesic to relive body aches and pain due to rheumatism and gout. It is also used to treat coughs, colds
and insect bite.
Reference: http://www.scribd.com/doc/19960697/10-approved-herbal-plants-by-DOH
35. Essential Maternal and Child Health Service Package includes the delivery of essential maternal and
child health and nutrition package of services that will ensure the right to survival, development,
protection and participation. The following are correct regarding Vitamin A supplementation among pregnant
and post-partum women except:
A. Vitamin A should not be given to pregnant woman who are already taking pre-natal vitamins that contain
Vitamin A.
B. Post-partum women should be given only a dose of 200, 000 IU of Vitamin A within 4 weeks after delivery.
C. A capsule of 10, 000 IU twice week is recommended for pregnant woman as soon as diagnosis of
pregnancy is done until delivery.

D. A and C only.
ANSWER: C
Rationale: Vitamin A supplementation for pregnant women should start at 4 months as the teratogenic effects to the
fetus may occur if given early during pregnancy. (DOH book, p.161)
Vitamin A Supplementation:
VITAMIN A
DOSE
SCHEDULE OF GIVING
REMARKS
Pregnant
10, 000 IU
2x a week starting on the 4th
Do not give vit. A supplementation before
women
month of pregnancy
the 4th month of pregnancy. It might
congenital problems in the baby.
Postpartum
200, 000 IU
1 cap; 1 dose only within 4
Vit. A of 200, 000 IU should not be given
women
weeks after delivery
to pregnant women.
Pregnant
10, 000 IU
1 cap/tab; 1x a day for 4 weeks Do not give vit. A 10, 000 IU if prenatal
women with
upon diagnosis
vitamins or multiple micronutrients tablets
nightblindness
containing vit. A are to be given. Vit. A can
be given regardless of AOG if pregnant
has nightblindness.
36. Also known as whooping cough, Pertussis is an acute and highly contagious infection of the respiratory
tract caused by Bordetella Pertussis. Pertussis means violent cough which describes the most
characteristic feature of the disease. Pikachu, 3 years old from a family of four siblings, a mother and a
father, was admitted to the hospital with a temperature of 101 F, a sore throat of four days duration, with difficulty
of swallowing and a moderate cervical adenitis. A diagnosis of nasopharyngeal diphtheria was made. Upon
browsing Pikachus chart, you read that the physician wrote in his assessment findings the term Bulls neck.
What does this medical term means?
A. Tender lymphnodes found at the anterior triangle of neck
B. Increase tone in the masseter muscles
C. Involuntary spasm of the neck muscles
D. Temporary swelling of the upper and lower extremities
ANSWER: A
RATIONALE: Tender lymphnodes found at the anterior triangle of neck
OPTION B- this is termed as trismus or lock jaw found in Tetanus
OPTION C- Involuntary spasm of the neck muscles is Nuchal Rigidity found in Meningitis.
OPTION D- Temporary swelling of the upper and lower extremities is called Lymphedema, a manifestation of
Filariasis.
Reference: Communicable eye in Communicable Diseases; An Easy Guide in Understanding Communicable
Diseases for Nurses by Roberto M. Salvador Jr., RN,MD,
37. Mumps is a disease caused by a virus that usually spreads through saliva and can infect many parts of the body,
especially the parotid salivary glands. Nurse Lester provides instructions to the mother of a child with mumps
regarding respiratory precautions. Mrs. V. Cruz asks the nurse about the length of time required for the
respiratory precautions. The nurse most appropriately responds:
A. Respiratory precautions are not necessary once the swelling appears
B. Respiratory precautions are not necessary before the swelling begins
C. Respiratory precautions are indicated for 18 days following the onset of parotid swelling
D. Respiratory precautions are indicated immediately before swelling and as long as there is glandular swelling
ANSWER: D
RATIONALE: The period of communicability for mumps begins before the glands are swollen and remains for an
unknown length of time, but it is presumed to last as long as any localized glandular swelling remains.

Mumps is transmitted via direct contact with or droplet spread from an infected person. Droplet precautions are
indicated during the period of communicability (immediately before and after swelling begins). Mumps is transmitted
through the contact with saliva of a infected person either droplets or by direct contact. The patient is infectious 6
days before the onset of parotid enlargement and 5 days after the appearance of parotitis. The incubation period is
14 to 25 days and it confers lifelong immunity even in unilateral mumps. Prognosis for complete recovery is
generally good.
38. Tuberculosis, or TB, is an infectious bacterial disease caused by Mycobacterium tuberculosis, which most
commonly affects the lungs. It is transmitted from person to person via droplets from the throat and lungs of
people with the active respiratory disease. Who among the following patients in the Barangay Espanya belongs
to Category 3 of the treatment regimen who are given 2 months RIPE during the intensive phase of treatment
and 4 months RI during the continuation phase?
A. Chronic Tuberculosis patient
B. Relapse
C. New smear (-) Pulmonary Tuberculosis patient with minimal parenchymal lesions on chest x-ray
D. New smear (-) Pulmonary Tuberculosis patient with extensive parenchymal lesions on chest x-ray
ANSWER: C
RATIONALE: Patients belonging to Category 3 are the new smear (-) Pulmonary Tuberculosis patient with minimal
parenchymal lesions on chest x-ray. The chronic TB patient belongs to category 4. The patient with the relapse
belongs to category 2. While the New smear (-) Pulmonary Tuberculosis patient with extensive parenchymal lesions
on chest x-ray belongs to category 1. (Reyala 2007 pp. 244-245)
39. In healthy people, infection with Mycobacterium tuberculosis often causes no symptoms, since the person's
immune system acts to wall off the bacteria. Direct Observed Treatment Short Course (DOTS) has been
advocated internationally to help control the spread of TB. The following are the element of DOTS except:
i. sustained political commitment
ii. access to quality-assured sputum microscopy
iii. standardized short-course chemotherapy for all cases of TB
iv. uninterrupted supply of quality assures drugs
v. recording and reporting system
A.
B.
C.
D.

i
ii
iii
None of the above

ANSWER: D.
RATIONALE: All of the things listed above are the element of TB DOTS. A better way to memorize is through using
the mnemonics CRASS. Commitment sustained politically, Recording and reporting system, Access to qualityassured sputum microscopy, Standardized short-course chemotherapy for all cases of TB and uninterrupted Supply
of quality assures drugs.
Reference: Reyala 2007 pp. 246-248
40. Tuberculosis is considered as the worlds deadliest disease and remains as a major public health
problem in the Philippines. A patient presents cough for more than 4 weeks, fever, and complains of night
sweats. He suffers from hemoptysis. Which of the following should be observed?
A. He shall be asked to undergo DSSM imemdiately.
B. CXR can be used to diagnose TB.
C. PPD skin test (Mantoux test) is the best option to diagnose TB.

D. None of the above.


ANSWER: D
RATIONALE: Since DSSM is the primary tool, no TB diagnosis shall be made based on the results of X-ray
examinations alone. Likewise, results of the skin test for TB infection (PPD skin test) should not be used as bases for
TB diagnosis in adults. The patient cannot undergo DSSM for he has hemoptysis. (DOH book)
41. Pertussis is diagnosed by its striking symptoms, although in children younger than 6 months of age, the whoop
of the cough may be absent, making it more difficult to diagnose. The B. pertussis bacillus may be cultured from
nasopharyngeal secretions during the catarrhal and paroxysmal stages. The white cell count, particularly the
lymphocyte count, increases with whooping cough: it may be as high as 20,000 to 30,000/mm3 at the end of the
catarrhal stage (normal is 5000 to 10,000/mm3). Adrian, an 8 year old boy, one of the several siblings in the
family, is kept home by his mother because he has a temperature of 100.2F and a history of hacking cough for
two weeks. A definite whoop has been heard and the doctor made a diagnosis of whooping cough. Whooping
cough is most contagious:
A. In the paroxysmal stage
B. In the febrile stage
C. In the catarrhal stage
D. In the incubation stage
ANSWER: C
RATIONALE:
Incubation Period is 7-10 days after exposure. The B. Pertussis will enter the tracheobronchial mucosa where it
produces progressively a tenacious mucus secretion. It is a prolonged illness with an average duration of 6-8 weeks
that includes three stages lasting for about 2 weeks each.
Stage
Length
Clinical Features
Stage 1:
Catarrhal

Usually 7-10 days;


range of 4-21

Stage 2:
Paroxysmal

Usually lasts 1-6


Characterized by:
weeks, but may
Paroxysms of numerous, rapid coughs due to difficulty expelling
persist for up to 10
thick mucus from the tracheobronchial tree.
weeks
Long aspiratory effort accompanied by a high-pitched "whoop" at
the end of the paroxysms
Cyanosis
Vomiting and exhaustion
Paroxysmal attacks:
Occur frequently at night, with an average of 15 attacks per 24
hours.
Increase in frequency during the first 1-2 weeks, remain at the
same frequency for 2-3 weeks, and then gradually decrease.

Stage 3:
Usually 7-10 days;
Convalescent range of 4-21

Characterized by:
Coryza
Low-grade fever
Mild, occasional cough (which gradually becomes more severe)

Characterized by:
Gradual recovery
Less persistent, paroxysmal coughs that disappear in 2-3 weeks
Paroxysms often recur with subsequent respiratory infections for many
months after the onset of pertussis.

Reference: Communicable eye in Communicable Diseases; An Easy Guide in Understanding Communicable


Diseases for Nurses by Roberto M. Salvador Jr., RN,MD.
42. Pertussis is a serious disease of childhood. Like diphtheria, pertussis has become quite rare because of required
immunizations, but it still occurs sporadically and is actually making a comeback in some locales and in
adolescents. Those most susceptible are children who have not been immunized because a previous vaccine
had possible side effects that led parents to refuse immunization. Nurse Adrian provides home care instructions
to the parents of a child hospitalized with pertussis. The child is in the convalescent stage and is being prepared
for discharge. Which statement by a parent indicates a need for further instructions?
A. We need to maintain droplet precautions and a quiet environment for at least 2 weeks.
B. We need to encourage our child to drink fluids.
C. Coughing spells may be triggered by dust or smoke.
D. Vomiting may occur when our child has coughing episodes.
ANSWER: A
RATIONALE: Pertussis is transmitted by direct contact or respiratory droplets from coughing. The communicable
period occurs primarily during the catarrhal stage. Respiratory precautions are not required during the convalescent
phase. Options A, B & C are components of home care instructions.
Reference: Communicable eye in Communicable Diseases; An Easy Guide in Understanding Communicable
Diseases for Nurses by Roberto M. Salvador Jr., RN,MD.
43. SARS or Severe Acute Respiratory Syndrome is a viral respiratory illness caused by a corona virus, called
SARS-associated corona virus (SARS-CoV), the same family of viruses that cause the common cold. These
viruses are a common caue of mild and moderate upper respiratory illness in humans and have occassionaly
been linked to pneumonia especially in people with weakened immune systems. . It was first reported in Asia in
February 2003. The following holds true about SARS, except?
A. SARS is potentially life threatening, with an average mortality rate of 11 percent for the most severely
affected patients.
B. SARS virus is spread by close person to person contact, either by air or touching materials contaminated
with SARS virus.
C. SARS may be suspected in a patient who has a history recent travel to the Western countries such as USA
or close contact with ill persons with a history of recent travel to such areas.
D. The incubation period of SARS is 2 to 7 days.
ANSWER: C
RATIONALE: SARS may be suspected in a patient who has:
Any of the signs and symptoms including a fever of 38 C or higher9cough, shortness of breath, difficulty
breathing, hypoxia, or radiographic findings of a pneumonia or acute respiratory syndrome
A history recent travel to China, HongKong, Singapore,Taiwan, Vietnam, Canada or close contact with
ill persons with a history of recent travel to such areas.
Is employed in an occupation at particular riskfor SARS exposure, healthcare worker with direct patient
contact or a worker in a laboratory that contain live SARS
OPTIONS B,C & D- are all correct statements
Reference: Communicable eye in Communicable Diseases; An Easy Guide in Understanding Communicable
Diseases for Nurses by Roberto M. Salvador Jr., RN,MD,
44. Infectious Mononucleosis is an acute infection of the respiratory tract that is caused by Epstein- Barr Virus
(EBR), a member of the herpes virus which is worldwide in distribution. It is also known as the Kissing Disease
because its mode of transmission is of intimate contact with an infected person. A home health nurse visits a

child with infectious mononucleosis and provides home care instructions to the parents about the care of the
child. The nurse tells the parents to:
A. Maintain the child on bed rest for 2 weeks.
B. Notify the physician if the child develops abdominal pain or left shoulder pain.
C. Maintain respiratory precautions for 1 week.
D. Notify the physician if the child develops a fever.
ANSWER: B
RATIONALE: The parents need to be instructed to notify the physician if abdominal pain, especially in the left upper
quadrant, or left shoulder pain occurs because this may indicate splenic rupture. Children with enlarged spleens also
are instructed to avoid contact sports until splenomegaly resolves. Bed rest is not necessary, and children usually
self-limit their activity. Respiratory precautions are not required, although transmission can occur via direct intimate
contact or contact with infected blood. Fever is treated with acetaminophen (Tylenol).
OPTIONS A & C- are unnecessary interventions in this disease
Reference: McKinney, E., James, S., Murray, S., & Ashwill, J. (2005). Maternal-child nursing (2nd ed., p. 1031). St.
Louis: W.B. Saunders.
45. It is important to document fetal presentation and position, because these help predict if the
presentation of a body part other than the vertex could be putting a fetus at risk. If a body part other than
the vertex presents to the cervix, labor is invariably longer because of ineffective descent of the fetus,
ineffective dilatation of the cervix, or irregular and weak uterine contractions. The labor and delivery nurse
performs Leopolds maneuvers. A hard round mass is felt in the fundal region. A flat object is noted on the left
and small objects are noted on the right of the uterus and a soft round mass is felt above the symphysis. Which
of the following positions is consistent with these findings?
A. Left occipital anterior (LOA)
B. Left sacral posterior (LSP)
C. Right mentum anterior (RMA)
D. Right sacral posterior (RSP)
ANSWER: B
RATIONALE: The findings after the nurse performs Leopolds maneuvers do not indicate that the fetus is in the left
sacral posterior (LSP) position; in that position, the fetuss buttocks (S or sacrum) are facing toward the mothers left
posterior (LP), a hard round mass is felt in the fundal region, and a soft round mass is felt above the symphysis.
OPTION A- The nurses findings upon performing Leopolds maneuvers indicate that the fetus is in the left occiput
anterior position (LOA)that is, the fetal back is felt on the mothers left side, the small parts are felt on her right side,
the buttocks are felt in the fundal region, and the head is felt above her symphysis.
OPTION C- The findings after the nurse performs Leopolds maneuvers do not indicate that the fetus is in the right
mentum anterior (RMA) position; in that position, the fetuss face (M or mentum) is facing toward the mothers right
anterior (RA) and small objects are felt on the right of the mothers abdomen with a flat area felt on the mothers left
side.
OPTION D- The findings after the nurse performs Leopolds maneuvers do not indicate that the fetus is in the right
sacral posterior (RSP) position; in that position, the fetuss sacrum (S) is facing the mothers right posterior (RP) and
a hard round mass is felt in the fundal region while a soft round mass is felt above the symphysis.
Reference: Maternal and Newborn Success: by Margot Desevo, 2013
46. Mrs. Lina has brought her 4-month old daughter to the RHU. She says that she is anxious that her child might
get sick again because of the vaccine. Nilagnat kasi ang anak ko at namula ung hita nya ng ilang araw
pagkatapos mabakunaan nung DPT. Pagkatapos ay kinumbulsyon pa sya isang linggo pagkatapos nung
bakuna. Hindi naman ata nakakatulong yan eh, nakakasakit pa. Tsaka nagtatae ung anak ko ngayon, kaya sabi

nung albularyo sa amin wag ko daw pabakuna. Pwede ba kaming bumalik na lang sa ibang araw? All of the
following are appropriate responses of Nurse Nessaj except for one:
A. Normal lang po na reaksyon ang lagnatin sa isang bata pagkatapos mabakunaan.
B. Maaari pa din po syang bakunaan ngayon kahit nagtatae, hindi na po kayo kelangang bumalik sa ibang
araw.
C. Hindi ko na po ibibigay ang pangalawang DPT nya dahil kinumbulsyon po sya.
D. Ang pamumula po ng hita pagkatapos mabakunaan ay normal na reaksyon ng bata sa bakuna.
ANSWER: C
RATIONALE: Fever, local reactions and systemic symptoms can result as a part of the normal immune response to
the vaccine. Moderate fever, malnutrition, mild respiratory function, cough, diarrhea and vomiting are not
contraindications to vaccinations. An absolute contraindication to DPT is that when the child has had convulsions or
shock within 3 days of the previous dose, DTP2 or DPT3 should never be given. (p. 143, Public Health Nursing in the
Phillpines by Cuevas, et.al., 10th edition, 2007)
47. No one can predict with certainty when labor will begin -- the due date your doctor gives you is merely a point of
reference. It is normal for labor to start as early as three weeks before that date or as late as two weeks after it.
A 16-year-old client at 38 weeks of gestation is admitted in labor. As the nurse takes the clients history, which
statement by the client indicates a risk factor?
A. I was surprised when my water broke. I didnt think it would be green.
B. The baby keeps moving. I didnt think it was supposed to move during labor.
C. I didnt realize my back would ache during contractions.
D. That bloody show has so much mucus in it!
ANSWER: A
RATIONALE: Green amniotic fluid indicates the passage of meconium, which suggests fetal distress and an
increased risk of meconium aspiration syndrome. Fetal movement before and during labor is normal; decreased or
increased fetal movement may be a sign of fetal distress. Contractions may be felt in the back, lower abdomen, or
both. Bloody show usually contains blood and mucus from the dislodged cervical mucus plug.
48. Uterine contractions and the pressure of the presenting part bring about cervical dilatation. Bloody show is the
first sign that the cervix is beginning to dilate. A primigravida client is showing signs of labor. Upon conducting
vaginal exam, the cervical dilatation at 10:00 am was 5 cm, and at 2 pm it was 8 cm. The correct interpretation of
this result is
A. The progress of labor is slow and abnormal
B. This is expected in the active phase
C. Labor is progressing normally
D. The client has entered the transition phase
ANSWER: C
RATIONALE: Specifically, primigravid women who enter the active phase at 3 to 4 cm reliably can be expected to
reach 8 to 10 cm dilation within 3 to 4 hours. This observation might have potential usefulness. For example, if
cervical dilatation reaches 4 cm, the clinician could expect complete dilatation to be achieved in approximately 4
hours if spontaneous labor is normal. (Williams Obstetrics by Cunningham 19th edition pp 478)
Latent phase: Mean: 8 hrs Max: 20 hours for nulliparous and Mean: 6 hrs Max: 14 hours for multiparous
Active phase: 1.2 cm/hr in nulliparous and 1.5 cm/hr for multiparous
49. The 6 Ps that affect the outcome and duration of labor are the Passage, Passenger, Person/Psyche,
Positioning, Placenta, and Power. Power includes the uterine contractions and maternal bearing down. The best

time to teach the client how to bear down is during the Latent phase, when the client is still comfortable. The
nurse would teach the woman to bear down
A. Only after full cervical dilatation
B. Only during contractions
C. In the absence of cardiac disease
D. All of the above
ANSWER:
RATIONALE: A.) Pushing during the Cervical stage of labor before full cervical dilatation can cause maternal fatigue,
which can lead to exhaustion during the Expulsive stage of labor when the mother already needs to push to expel the
fetus. Exhaustion can lead to forceps delivery, difficult labor, and fetal compromise. B.) The voluntary power
(maternal bearing down) and involuntary power (uterine contractions) should act together in order to be more
effective. C.) Clients with heart conditions (whether male or female) are not encouraged to perform valsalva
maneuver (ex., pushing, straining, etc.) because it can cause vagal stimulation, which could cause bradycardia.
Women with heart conditions deliver via forceps delivery to prevent pushing in normal spontaneous delivery and to
prevent blood loss in caesarian delivery.
50. In preparation for labor and delivery, the pregnant woman must be taught how to determine the events that
signal the beginning of labor. Rupture of Membranes is one of these events which predispose a mother and her
child at risk for infection. Which assessment findings signal the rupture of membranes?
i. sudden gush of clear liquid
ii. Nitrazine paper turned into red
iii. Nitrazine paper turned into blue green
iv. ferning is present upon examination under a microscope
v. no ferning is present upon examination under a microscope
A.
B.
C.
D.

i only
i, iii, iv
i, ii, v
i, iii, v

ANSWER: B
RATIONALE: The following are the signals of ROM: sudden gush of fluid from the vagina with continued minimal
leakage which may be mistaken with urine; nitrazine paper causes an alkaline reaction which turns into blue or
green, gray or deep blue; ferning occurs due to the high estrogen content of the amniotic fluid. (Pilliteri Vol. 1, pp.
423-514)
51. Amnioinfusion refers to the addition of sterile fluid to the amniotic fluid. Mrs. Cruz is scheduled for amnioinfusion
after evaluation through external fetal monitor. Her baby is having variable deceleration of FHR which shows
cord compression. As the nurse assigned to her, Nurse Lester correctly assisted the physician with the
procedure when she does the following except:
A. helps prevent infecting the site by following the aseptic technique protocols of the institution
B. obtains the vital signs routinely, especially the temperature of the Mrs. Lauren
C. monitors the fetal heart rate and uterine contraction just once
D. none of the above
ANSWER: C
RATIONALE: The nurse should help maintain strict aseptic technique during insertion and while caring for the
catheter. The fetal heart rate and uterine contraction should be continuously monitored. Monitor the temperature
hourly to help ascertain if there already is an infection. (Pilliteri Vol. 1, pp. 526-527)

Decelerations are temporary drops in the fetal heart rate. There are three basic types of decelerations: early
decelerations, late decelerations, and variable decelerations.
Early Decelerations: are seen when the baby's head is compressed. This often happens during the later stages of
labor as the baby is descending through the birth canal. They may also occur during early labor if the baby is
premature or in a breech position, causing its head to be squeezed by the uterus during contractions. Early
decelerations have absolutely no clinical significance and are not harmful. The nadir occurs with the peak of a
contraction.
Late decelerations do not begin until the peak of a contraction or thereafter. They are smooth, shallow dips in heart
rate that mirror the shape of the contraction that is causing them. Onset of the deceleration occurs after the beginning
of the contraction, and the nadir of the contraction occurs after the peak of the contraction. Late decelerations are
among the most worrisome form of heart rate patterns because they usually signify a reduced oxygen supply to the
baby.
Management of Late Decelerations : These maneuvers are primarily intended to alleviate "reflex"
lates.
Place patient on side [23,24]
Discontinue oxytocin.
Correct any hypotension
IV hydration.
If decelerations are associated with tachysystole consider terbutaline 0.25 mg SC [26,27]
Administer O2 by tight face mask [25, 40]
If late decelerations persist for more than 30 minutes despite the above maneuvers, fetal scalp pH is
indicated.
Scalp pH > 7.25 is reassuring, pH 7.2-7.25 may be repeated in 30 minutes.
Deliver for pH < 7.2 or minimal baseline variability with late or prolonged decelerations and inability to
obtain fetal scalp pH [28,29]
Variable Decelerations do not look like late or early decelerations. They are generally irregular, often jagged dips in
the fetal heart rate that look more dramatic than late decelerations. Variable decelerations are caused when the
umbilical cord of the baby is temporarily compressed. As a rule, variable decelerations are not worrisome. However,
the baby does depend upon steady blood flow through the umbilical cord to receive oxygen and other important
nutrients. If variable decelerations are prolonged or repetitive, they can signify a reduction of blood flow, which is
harmful to the baby.
Management of Variables
Change position to where FHR pattern is most improved. Trendelenburg may be helpful.
Discontinue oxytocin.
Check for cord prolapse or imminent delivery by vaginal exam.
Consider amnioinfusion[35-37]
Administer 100% O2 by tight face mask [4].
Reference: http://www.perinatology.com/Fetal%20Monitoring/Intrapartum%20Monitoring.htm
52. The most accurate measure of the progress of labor is cervical dilatation and effacement. Prior to full cervical
dilatation, what other signs and symptoms would support that true labor is happening?
A. Pain begins at the lower back and radiates to the abdomen
B. Walking makes the contractions longer
C. Contractions get stronger, longer, and closer together regardless of position
D. All of the above

ANSWER: D
RATIONALE:
True Labor
Contractions occur at
regular intervals, last 30 to
60 seconds.

False Labor
Contractions usually not
regular.

What To Do
If you think you are in labor:

Contractions get gradually


closer together and
stronger.

Contractions do not get


closer together; may stop
after an hour or so.

Time your contractions with a watch. (If


one contraction starts at 9:10 and the
next starts at 9:16, then they are 6
minutes apart)

Contractions may be felt in


your back and lower
abdomen.

Contractions not usually felt


in your back

Contractions continue or
increase with walking.

Contractions stop with


walking.

There is increasing
discomfort.

Discomfort does not


increase.

Your cervix softens,


shortens, and dilates.

Your cervix does not change.

Show is present in small to


moderate amount due to
cervical changes.

Show is not present.

Tell your doctor or clinic. Wear a pad if


you your wish.

Bag of waters may break


before you feel
contractions.
Fluid usually continues to
leak, especially during a
contraction, which may
become stronger.

Bag of water does not break.

If you think your water has broken, call


your doctor or healthcare provider right
away and go to the hospital.
Do not take a bath or put anything into
your vagina.
Wear a pad, if necessary, until you
arrive at the hospital.

Rest, if possible, on your side.

Eat or drink only light foods.


Come to the hospital when your
contractions are 5 to 6 minutes apart.

53. The station denotes the degree of engagement of the fetal head as it navigates the maternal pelvis. The
landmark for determining the Station is the maternal ischial spine. Which of the following is INCORRECT about
the different Stations?
A. At station 0, the fetal head engages in the maternal pelvis, which could happen 2 weeks before labor
B. Clients may show no changes in station in case of cephalopelvic disproportion even with full cervical
dilatation
C. After station 0, the client feels the lesser pressure and pain on the pelvic bone and vaginal area
D. At (+) 3 to (+) 5, the babys head is crowning
Station refers to how far down the baby's head has descended into the mother's pelvis. The birth canal is shaped
like a cone, curving underneath and behind the pubic bone. Its narrowest section lies between two bony landmarks of
the pelvis (one on either side), called the ischial spines.

If the presenting part lies above the ischial spines, the station is reported as a negative number from -1 to -5 (each
number is a centimeter). If the presenting part lies below the ischial spines, the station is reported as a positive
number from +1 to +5. The baby is said to be "engaged" in the pelvis when it is even with the ischial spines at 0
station.
54. Small lacerations or tears of the birth canal are common and may be considered a normal consequence of
childbearing. Large lacerations, however, can cause complications. During the delivery of a macrosomic baby,
the woman develops a fourth-degree laceration. How should the nurse document the extent of the laceration in
the womans medical record?
A. Through the rectal sphincter.
B. Into the musculature of the buttock.
C. Through the urinary meatus.
D. Into the head of the clitoris.
ANSWER: A
RATIONALE: A fourth-degree laceration extends through the rectal sphincter. One of the many complications
that can occur with the delivery of a macrosomic baby is a perineal laceration. If the laceration is extensive
and it progresses through the rectal sphincter it is defined as a fourth degree. As a result, this client is at high risk
for the development of a vaginal-rectal fistula.
OPTION B- laceration into the musculature of the buttocks is defined as a second-degree laceration.
OPTION C- A fourth-degree laceration extends through the rectal sphincter.
OPTION D- A fourth-degree laceration extends through the rectal sphincter.
Degree of Perineal Laceration:
1. First-degree vaginal tears are the least severe, involving only the skin around the vaginal opening. To ease any
discomfort during urination, pour warm water over your vulva as you're passing urine.
2. Second-degree vaginal tears involve vaginal tissue and the perineal muscles the muscles between the
vagina and anus that help support the uterus, bladder and rectum. Second-degree tears typically require stitches
and heal within a few weeks.
To ease discomfort while you're recovering:
Sit on a pillow or padded ring.
Pour warm water over your vulva as you're passing urine, and rinse yourself with a squeeze bottle
afterward. Press a clean pad firmly against the wound when you bear down for a bowel movement.
Cool the wound with an ice pack, or place a chilled witch hazel pad between a sanitary napkin and the
wound.
Take pain relievers or stool softeners as recommended by your health care provider.
3. Third-degree vaginal tears involve the vaginal tissues, perineal muscles and the muscle that surrounds the anus
(anal sphincter). These tears sometimes require repair in an operating room rather than the delivery room
and might take months to heal. Complications such as fecal incontinence and painful intercourse are possible.
To ease discomfort in the meantime:
Sit on a pillow or padded ring.
Pour warm water over your vulva as you're passing urine, and rinse yourself with a squeeze bottle
afterward. Press a clean pad firmly against the wound when you bear down for a bowel movement.
Cool the wound with an ice pack, or place a chilled witch hazel pad between a sanitary napkin and the
wound.
Take pain relievers or stool softeners as recommended by your health care provider.
4. Fourth-degree vaginal tears are the most severe. They involve the perineal muscles and anal sphincter as well
as the tissue lining the rectum. Fourth-degree tears usually require repair in an operating room rather than the
delivery room and might take months to heal. Complications such as fecal incontinence and painful
intercourse are possible.

Reference: Maternal and Newborn Success: by Margot Desevo, 2013


55. Assessing fetal growth throughout pregnancy, by such means as fundal height and fetal heart rate, is important
because these predictable signs of fetal development provide guides for determining the well-being of fetuses.
For the expectant family, knowledge about fetal growth and development can provide an important frame of
reference, helping a mother to understand some of the changes going on in her body and allowing all family
members to begin thinking about and accepting the newest member of the family before the baby is actually
born. You are the nurse assigned to care for a 30-week-gestation client whose fetal fibronectin (fFN) levels are
positive. It is essential that she should be inculcated about which of the following?
A. How to use a blood glucose monitor.
B. Signs of preeclampsia.
C. Signs of preterm labor.
D. How to do fetal kick counts.
ANSWER: c
RATIONALE: Fetal fibronectin (fFN) is a substance that is metabolized by the chorion. Although positive during the
first half of pregnancy, it is very rare to see positive results between 24 and 34 weeks gestation unless the clients
cervix begins to efface and dilate. It is an excellent predictor of preterm labor (PTL); therefore, many practitioners
assess the cervical and vaginal secretions of women at high risk for PTL for the presence of fFN. Positive fetal
fibronectin levels are seen in clients who deliver preterm.
OPTION A- Fetal fibronectin is not related to glucose metabolism.
OPTION C- Fetal fibronectin is not related to hypertensive conditions.
OPTION D- Fetal fibronectin is not related to fetal distress.

To avoid a false-positive result, the test will be done before any pelvic exam or vaginal ultrasound. It's also best
to avoid having sex for 24 hours before the test. Vaginal bleeding can affect the test results. If you experience
vaginal bleeding, the fetal fibronectin test will likely be postponed.
During the fetal fibronectin test, you'll lie on your back on an exam table. Your health care provider will place a
speculum in your vagina and use a cotton swab to gently swipe secretions near the cervix. The sample will be
sent to a lab for analysis. Results are typically available within less than 24 hours.
Results: Positive. A positive result means that fetal fibronectin is present. If you have a positive result between
weeks 22 and 34, you're at increased risk of preterm labor.
Negative. A negative result means that fetal fibronectin isn't present. This indicates that your pregnancy is likely
to continue for at least another week.
Warning signs of preterm labor include:
Regular or frequent contractions a tightening sensation in the abdomen
Constant low, dull backache
A sensation of pelvic or lower abdominal pressure
Mild abdominal cramps
Diarrhea
Vaginal spotting or bleeding
Watery vaginal discharge (water breaking) in a gush or a trickle
A change in vaginal discharge
Reference: Maternal and Child Health Nursing: Care of the Childearing and Childrearing Family by Adele Pilliterri,
Lippincott Williams & Wilkins, 2007
56. Periodic changes or fluctuations in FHR occur in response to contractions and fetal movement and are described
in terms of accelerations or decelerations. Periodic changes are short term changes in rate rather than baseline;
they last from a few seconds to 1 or 2 minutes. While caring for a client in the second stage of labor, you noticed
a pattern of early decelerations. As a prudent nurse, you should:
A. Notify the physician immediately

B. Document the finding on the flow sheet


C. Turn the client on her left side
D. .Apply oxygen via a tight face mask
ANSWER: B
RATIONALE: Early decelerations are normal periodic decreases in FHR resulting from pressure on the fetal head
during contractions. Parasympathetic stimulation in response to vagal nerve compression brings about a slowing of
FHR. Early deceleration follows the pattern of the contraction, beginning when the contraction begins and ending
when the contraction ends. However, the waveform of the FHR change is the inverse of the contraction waveform,
with the lowest point of the deceleration occurring with the peak of the contraction. In this way, it serves as a mirror
image of the contraction. The rate rarely falls below 100 bpm, and it returns quickly to between 120 and 160 beats at
the end of the contraction. Early decelerations normally occur late in labor, when the head has descended fairly low.
As such, they are viewed as a normal pattern. However, if they occur early in labor, before the head has fully
descended, the head compression causing the waveform change could be the result of cephalopelvic disproportion
and is a cause to investigate. Early decelerations during the second stage of labor are benign and are the result of
fetal head compression that occurs during normal contractions. No action is necessary other than documenting the
finding on the flow sheet.
OPTIONS A, C, and D - are interventions for the client with late decelerations, which reflect ureteroplacental
insufficiency.
Reference: Maternal and Child Health Nursing: Care of the Childearing and Childrearing Family by Adele Pilliterri,
Lippincott Williams & Wilkins, 2007
57. Epidural anesthesia involves injection of drugs through a catheter placed into the epidural space. For a
prolonged effect, a continuous infusion of drugs may be employed. Which of the following would be an
unnecessary intervention for a post-partum client who has received epidural anesthesia?
A. Assessing the clients ability to void within 6 hours after delivery
B. Allowing the woman to move around in bed
C. Maintaining the client on supine position for 8-12 hours
D. Encouraging the woman to ambulate upon return of sensation and strength
ANSWER: C
RATIONALE: Epidural anesthesia is regional anesthesia that blocks pain in a particular region of the body. The goal
of an epidural is to provide analgesia, or pain relief, rather than anesthesia which leads to total lack of feeling. This
results in decreased sensation in the lower half of the body. After an epidural, the catheter will be removed, and you
will lie in bed until you have feeling in your legs and can walk.
ADVANTAGES:
Allows you to rest if your labor is prolonged
By reducing the discomfort of childbirth, some woman have a more positive birth experience
Normally, an epidural will allow you to remain alert and be an active participant in your birth
If you deliver by cesarean, an epidural anesthesia will allow you to stay awake and also provide effective pain
relief during recovery
When other types of coping mechanisms are no longer helping, an epidural can help you deal with exhaustion,
irritability, and fatigue. An epidural can allow you to rest, relax, get focused and give you the strength to move
forward as an active participant in your birth experience.
The use of epidural anesthesia during childbirth is continually being refined and much of its success depends on
the skill with which it is administered.
DISADVANTAGES:
Epidurals may cause your blood pressure to suddenly drop. For this reason your blood pressure will be routinely
checked to help ensure an adequate blood flow to your baby. If there is a sudden drop in blood pressure, you
may need to be treated with IV fluids, medications, and oxygen.

You may experience a severe headache caused by leakage of spinal fluid. Less than 1% of women experience
this side effect. If symptoms persist, a procedure called a blood patch, which is an injection of your blood into
the epidural space, can be performed to relieve the headache.
After your epidural is placed, you will need to alternate sides while lying in bed and have continuous monitoring
for changes in fetal heart rate. Lying in one position can sometimes cause labor to slow down or stop.
You might experience the following side effects: shivering, ringing of the ears, backache, soreness where the
needle is inserted, nausea, or difficulty urinating.
You might find that your epidural makes pushing more difficult and additional interventions such as Pitocin,
forceps, vacuum extraction or cesarean might become necessary
For a few hours after the birth the lower half of your body may feel numb. Numbness will require you to walk with
assistance.
In rare instances, permanent nerve damage may result in the area where the catheter was inserted.
Though research is somewhat ambiguous, most studies suggest that some babies will have trouble latching on
causing breastfeeding difficulties. Other studies suggest that a baby might experience respiratory depression,
fetal malpositioning, and an increase in fetal heart rate variability, thus increasing the need for forceps, vacuum,
cesarean deliveries and episiotomies.
NOTES:
Some women describe an epidural placement as creating a bit of discomfort
Typically epidurals are placed when the cervix is dilated to 4-5 centimeters and you are in true active labor.
Your epidural can cause your labor to slow down and make your contractions weaker. If this happens you may
be given the medicine Pitocin to help speed up labor.
You might not be able to tell that you are having a contraction because of your epidural anesthesia. If you can
not feel your contractions, then pushing may be difficult to control
58. The primary purpose of episiotomy is to enlarge the vaginal opening and assist childbirth. It is done with local
anesthesia and then sutured after delivery. Episiotomy is performed
A. While the client is bearing down
B. Before crowning to prepare the passage
C. While Ritgens maneuver is performed to extensive lacerations
D. When the head is crowning
ANSWER: D
RATIONALE: Just before your baby is born, and as the head is about to crown, your doctor or midwife will give you a
shot to numb the area (if you havent already had an epidural). Next, a small incision (cut) is made. There are two
types of cuts: median and medio-lateral.
1. The median incision is the most common type. It is a straight cut in the middle of the perineum.
2. The medio-lateral incision is made at an angle. It is less likely to tear through to the anus, but it takes longer to
heal than the median cut.

Times when an episiotomy is often performed include:


If you are pushing as the babys head is close to coming out, and you tear up toward the urethral area
If labor is stressful for the baby and the pushing phase needs to be shortened to decrease problems for the baby
If the baby's head or shoulders are too big for the mother's vaginal opening
If the baby is in a breech position (feet or buttocks coming first) and there is a problem during delivery
If instruments (forceps or vacuum extractor) are needed to help get the baby out

59. APGAR scoring parameters includes: heart rate, respiratory effort, muscle tone, reflex irritability, and color. Upon
reviewing the chart of a newborn, Nurse Dhang found out that the baby has an APGAR score of 5 for 5-minute.
She knows that:
A. The newborn is in danger and needs immediate resuscitation
B. The newborns condition is guarded and may need clearing the airway and supplementary oxygen.

C. The newborns condition is guarded and needs immediate resuscitation.


D. The newborn is in good condition.
ANSWER: B
RATIONALE: A score is given for each sign at one minute and five minutes after the birth. If there are problems with
the baby an additional score is given at 10 minutes. A score of 7-10 is considered normal, while 4-7 might require
some resuscitative measures, and a baby with apgars of 3 and below requires immediate resuscitation.
60. At 1 minute and 5 minutes after birth, newborns are observed and rated according to an APGAR score. Heart
rate, respiratory effort, muscle tone, reflex irritability, and color are among the 5 parameters. Nurse Bhum
documents the findings to a male newborn: HR 101bpm, slow cry, some flexion of extremities, flex arms and
legs in reaction to a slap to the sole of the foot and is pink in appearance. The newborns APGAR score is:
a. 5
b. 6
c. 7
d. 8

61. Cesarean birth, or birth accomplished through an abdominal incision into the uterus, is one of the oldest
types of surgical procedures known. It is always slightly more hazardous than vaginal birth, but
compared with other surgical procedures, it is one of the safest types of surgeries and one with few
complications. Nurse Cleofe has been assigned to circulate during the cesarean section of triplets. As a
knowledgeable nurse, you know that which among the following actions should Nurse Cleofe not take before the
birth of the babies?
A. Count the number of sterile sponges.

B. Document the time of the first incision.


C. Notify the pediatric staff.
D. Assemble the sterile instruments.
RATIONALE:
OPTION D- It is not appropriate for the circulating nurse to assemble the sterile instruments since he or she is not
sterile. The circulating nurse is responsible for coordinating the activity in the operating room. He or she is the
only member of the team who is able to move freely throughout the room in order to make telephone calls, obtain
needed supplies, maintain the documentation record, and so on. When multiple babies are being birthed, he or she is
especially important. The more babies who are birthed at oncee.g., twins, tripletsthe more vulnerable the babies
are at birth. Multiple-gestation babies are often born preterm and small-forgestational age. There must be a
resuscitation team available for each baby in case emergent care is needed.
OPTION A- The circulating nurse should count the sterile sponges. This is done together with the scrub nurse.
OPTION B- The circulating nurse must document in the medical record all key events that occur during the surgery,
including the time of the first incision.
OPTION C- The circulating nurse should notify the pediatric staff. There should be one resuscitation team assembled
in the delivery room for each baby that will be delivered.
Reference: Maternal and Newborn Success: by Margot Desevo, 2013
62. Special treatment needs of the newborn should be assessed by reviewing the history of labor and delivery and
the mothers condition for infections during pregnancy. You are caring for a 1 day old newborn that has a history
of ruptured membranes 24 hours before her delivery. You will classify this baby under:
A. Risk for Bacterial Infection
B. Risk for Congenital Syphillis
C. Risk for HIV Transmission
D. Risk for Local Umubilical Infection

Possible Serious Illness


Risk of Bacterial infection
Risk of congenital Syphilis
Risk of TB

- Ampicillin + Gentamicin before referral


Ampicillin + Gentamicin 5 days
Benzathine Penicillin (1 dose)
- Isoniazid X 6 months- BCG vaccination AFTER
(1 dose)

Possible Gonococcal Eye infection


- 1st choice: Ceftriaxone
- 2nd choice: Kanamycin
Mastitis
- Cloxacillin X 10 days
Eye infection
- Tetracycline TID
Local infection
- Gentian violet TID X 5 days

63. Phimosis in infancy is nearly always physiological, and needs to be treated only if it is causing obvious problems
such as urinary discomfort or obstruction. Mrs. Cruz brought her 2 year old son with phimosis to the Out Patient
Department for consultation. She informed you that her son is experiencing recurrent infection and asks you
what would be the management of choice?
A. To delay circumcision
B. Early circumcision
C. HCG administration
D. To allow the prepuce to grow
ANSWER: B
RATIONALE: Choice of treatment is often determined by whether the patient (or doctor) views circumcision as an

option of last resort to be avoided or as the preferred course. Phimosis persisting after the age of two years may be
considered for further treatment, particularly if recurrent infections of the foreskin (balanoposthitis) or urinary tract
infections are occurring. The options are plastic surgery or circumcision (http://www.patient.co.uk/doctor/phimosisand-paraphimosis)

Phimosis is tightness of the prepuce (foreskin) of the penis that prevents the retraction of the foreskin over the
glans. The condition is usually congenital but it may be the result of an infection.

1. Physiologic phimosis: Children are born with tight foreskin at birth and separation occurs naturally over time.
Phimosis is normal for the uncircumcised infant/child and usually resolves around 5-7 years of age, however the
child may be older.
2. Pathologic phimosis: Phimosis that occurs due to scarring, infection or inflammation. Forceful foreskin retraction
can lead to bleeding, scarring, and psychological trauma for the child and parent. If there is ballooning of the
foreskin during urination, difficulty with urination, or infection, then treatment may be warranted.
Treatments for phimosis vary depending on the child and severity of phimosis. Treatments may include: gentle
daily manual retraction, topical corticosteroid ointment application or circumcision.
Reference: http://urology.ucsf.edu/patient-care/children/phimosis
64. A child with cystic fibrosis has a genetic fault that causes his body to make mucus that is so heavy and sticky
that it blocks the lungs and pancreas, making breathing and digestion difficult. When developing a plan of care
for a child with cystic fibrosis, the nurse would perform postural drainage at which of the following times?
A. Early morning after breakfast
B. After meals
C. Before meals
D. After rest periods
ANSWER: C
RATIONALE: Postural drainage, which aids in mobilizing the thick, tenacious secretions is usually performed before
meals to avoid possibility of vomiting or regurgitating food. Loosening the thick mucus in the lungs makes it easier to
cough up. Chest physical therapy helps loosen mucus and is usually done from one to four times a day. A common
technique is clapping with cupped hands on the front and back of the chest (http://www.mayoclinic.org/diseasesconditions/cystic-fibrosis/basics/treatment/con-20013731)

The thick and sticky mucus associated with cystic fibrosis clogs the tubes that carry air in and out of your lungs.
This can cause:
A persistent cough that produces thick spit (sputum) and mucus
Wheezing
Breathlessness
A decreased ability to exercise
Repeated lung infections
Inflamed nasal passages or a stuffy nose
The primary goals of CF treatment include the following:
Maintaining lung function as near to normal as possible by controlling respiratory infection and clearing airways
of mucus
Administering nutritional therapy (ie, enzyme supplements, multivitamin and mineral supplements) to maintain
adequate growth
Managing complications
Mild acute pulmonary exacerbations of cystic fibrosis can be treated successfully at home with the following
measures:
Increasing the frequency of airway clearance
Inhaled bronchodilator treatment
Chest physical therapy and postural drainage
Increasing the dose of the mucolytic agent dornase alfa (Pulmozyme)
Use of oral antibiotics (eg, oral fluoroquinolones)
Reference:
Maternal
and
Child
Health
Nursing
by
Pillitteri,
4th
edition;
http://emedicine.medscape.com/article/1001602-treatment
65. Coarctation of Aorta is a congenital narrowing of part of the aorta. A toddler has been diagnosed as having
coarctation of the aorta. Considering the childs diagnosis, the nurse should expect which of these findings?
A. Bounding femoral pulse
B. Machinery-like murmur
C. Weak, thready radial pulse
D. Blood pressure higher in upper extremities
ANSWER: D
RATIONALE: Coarctation of the aorta or aortic coarctation is a narrowing of the aorta, the large blood vessel
that branches off your heart and delivers oxygen-rich blood to your body. When this occurs, your heart must pump
harder to force blood through the narrow part of your aorta.
The diagnosis of coractation of the aorta may be made on the grounds of the history and physical assessment. On
examination, the blood pressure in the arms will be at least 20 mm Hg higher than in the legs, a reversal of the
normal pattern. The blood pressure in the legs is usually weaker than in the arms. Blood pressure is usually higher in
the arms after infancy.
OPTION A: The pulse in the groin (femoral) area or feet will be weaker than the pulse in the arms or neck (carotid).
Sometimes, the femoral pulse may not be felt at all.
OPTION B: Machinery-like murmur is seen in patients with PDA. People with aortic coarctation have a harshsounding murmur that can be heard from the back.
OPTION C: Since there is high blood pressure in the upper extremities, bounding radial pulse is also expected.

Reference: Maternal and Child Health Nursing by Pillliteri 4th edition pp 1251; http://www.mayoclinic.org/diseasesconditions/coarctation-of-the-aorta/basics/symptoms/con-20031772;
http://www.nlm.nih.gov/medlineplus/ency/article/000191.htm
66. Hypospadias is a term for a urethral opening located on the inferior or ventral (under) surface of the penis. This
anomaly is fairly common, occurring in approximately 1 in 300 male newborns. It tends to be familial or may
occur from a multifactorial genetic focus. Baby Jigs, 18-month-old is being discharged following hypospadias
repair. Which instruction should be included in the nurses discharge teaching?
A. Applying warm compresses will decrease pain.
B. The child should not play on his rocking horse.
C. Diapering should be avoided for 12 weeks.
D. The child will need a special diet to promote healing.
ANSWER: B
RATIONALE: Following hypospadias repair, the child will need to avoid straddle toys, such as a rocking horse, until
allowed by the surgeon. Because the hospital stay typically is less than 24 hours, provide discharge teaching
expeditiously.
Demonstrate how to care for urinary discharge system, paying particular attention to the tube anchoring and
patency.
Encourage the parents to provide foods high in protein and fiber with extra fluids.
Educate about prescribed medications
Instruct parents how to care for post operative wound edema and how ecchymosis eventually resolves.
Explicitly describe activities that need to be curtailed until healing is complete (usually 2-3 weeks) such as rough
play, swimming and contact sports. Emphasize that the child must not play on any straddle type toys for at least
2-3 weeks.
Review bathing techniques as prescribed by the surgeon.
OPTION A- The child may notice painful bladder spasms as long as the catheter is in place (3 to 7 days). An
analgesic such as acetaminophen (Tylenol) and an antispasmodic medication such as oxybutynin (Ditropan)
may be prescribed for pain relief.
OPTION C- The drainage tube is connected either in a closed drainage unit or in an open system in which case a
double diaper technique is used to shield the operative site from moisture, fungal infections and skin breakdown .
With this technique, the open drainage is sandwiched between two diapers. The bottom diaper covering the penis,
remains dry.
OPTION D- No specific strict diet is prescribed for post operative case for hypospadias.
Reference: Children and Their Families: The Continuum of Care By Vicky R. Bowden, Cindy Smith, Lippincott
Williams & Wilkins, 2010

67. Nurse Jerry obtained his patients laboratory results. He found that his patients serum calcium level is
7.2 mg/dL. Of the following assessment data obtained from his patient, which of the following will the
Nurse Jerry consider to be LEAST related to the laboratory results?
a. Muscle weakness
b. Tingling sensations around the mouth
c. Abdominal cramping
d. Decrease blood pressure
ANSWER: C
Rationale: The normal Ca level is 8.5- 10.5 mg/dL. Abdominal pain or abdominal cramping is a sign of
hypercalcemia.
Options A, B, and D are signs of hypocalcemia. The earliest sign are paresthesias which come in the form
of cicumoral paresthesias.

68. The two types of electrolyte imbalances are deficit and excess. In general, if a patient experiences a
deficit of an electrolyte, the electrolyte is replaced either orally or intravenously. If the patient
experiences an excess of the electrolyte, treatment focuses on getting rid of the excess, often by the
kidneys. The underlying cause of the imbalance must also be treated. The most important aspect of
nursing care is preventing and assessing electrolyte imbalances. During your shift, Mang Gin, 50 year old
client presents to the emergency department (ED) with tachycardia, elevated blood pressure, and seizures.
Further assessment reveals a history of chronic alcoholism, causing the nurse to suspect:
a. Magnesium deficit.
b. Sodium deficit.
c. Potassium excess.
d. Calcium excess.
RATIONALE:
OPTION A- Alcoholics tend to have hypomagnesemia as their primary electrolyte imbalance due to a poor dietary
intake. Hypomagnesemia occurs when the serum magnesium level falls below 1.5 mEq/L. It results from either a
decreased intake or an excessive loss of magnesium. Causes of inadequate intake include malnutrition and
starvation diets. Patients with severe diarrhea and Crohns disease are unable to absorb magnesium in the
intestines.One of the major causes of hypomagnesemia is alcoholism, which causes both a decreased intake and an
increased renal excretion of magnesium. Certain drugs, such as loop (high-ceiling) and osmotic diuretics,
aminoglycosides (e.g., gentamicin), and some anticancer agents (e.g., cisplatin), can increase renal excretion of
magnesium. The signs and symptoms of hypomagnesemia are similar to those for hypocalcemia, including positive
Trousseaus and Chvosteks signs.
OPTION B- A sodium deficit can cause seizures, but the other symptoms are not consistent with hyponatremia. The
signs and symptoms of hyponatremia are vague and depend somewhat on whether a fluid imbalance accompanies
the hyponatremia. The patient with sodium and fluid deficits has signs and symptoms of dehydration. The patient with
a sodium deficit and fluid overload has signs and symptoms associated with fluid overload. In addition, the patient
experiences mental status changes, including disorientation, confusion, and personality changes, caused by cerebral
edema (fluid around the brain). Weakness, nausea, vomiting, and diarrhea may also occur.
OPTION C- The symptoms in the question are not consistent with hyperkalemia. Most cases of hyperkalemia occur
in patients who are hospitalized or those undergoing medical treatment for a chronic condition. The classic
manifestations are muscle twitches and cramps, later followed by profound muscular weakness; increased GI motility
(diarrhea); slow, irregular heart rate; and decreased blood pressure. Cardiac dysrhythmias and respiratory
failure can occur in severe hyperkalemia, causing death.
OPTION D- Hypercalcemia weakens muscles, and therefore would not cause seizures. Patients who have mild
hypercalcemia or a slowly progressing calcium increase may have no obvious signs and symptoms. However, acute
hypercalcemia is associated with increased heart rate and blood pressure, skeletal muscle weakness, and decreased
GI motility. The patient also has a decreased blood clotting capability.In some cases the patient may experience
renal or urinary calculi (stones) resulting from the buildup of calcium. In more severe cases of acute hypercalcemia,
the patient may experience respiratory failure caused by profound muscle weakness or heart failure caused by
dysrhythmias.
REFERENCE: Understanding Medical-Surgical Nursing , Second Edition, by Linda S. Williams, 2003 by F. A. Davis
Company
69. Most acid-base imbalances are caused by a number of acute and chronic illnesses or conditions. The primary
treatment for each of the imbalances is to manage the underlying cause, which corrects the imbalance. The role
of the nurse is to identify patients at risk and monitor laboratory test values for significant changes. When
performing an assessment of a client admitted with metabolic alkalosis, the nurse should ask about the use of?
A. Aspirin.
B. Antacids.
C. Acetaminophen.

D. Antihistamines.
ANSWER: B
RATIONALE: Metabolic alkalosis is a clinical disturbance characterized by a high pH (decreased H+ concentration)
and a high plasma bicarbonate concentration. It can be produced by a gain of bicarbonate or a loss of H+
OPTION B- Antacids are very alkaline, and too many of them can cause metabolic alkalosis.
OPTION A- Aspirin overdose initially causes respiratory alkalosis as ASA increases respirations, but over time
metabolic acidosis can occur as ASA is acidic.
OPTION C & D- acetaminophen and antihistamine overdose can lead to metabolic acidosis
Metabolic acidosis
CAUSES
Diabetic ketoacidosis,
malnutrition, starvation
Lactic acidosis

HOW THEY AFFECT ACID BASE BALANCE


The body breaks down fat for energy, producing
the acid ketones. Ketones are a byproduct of fat metabolism. Ketones are acids!
Arterial disorders decrease oxygenated blood in the tissues. This causes the body to
switch from aerobic metabolism (using oxygen) to anaerobic metabolism (without
oxygen). The end product of anaerobic metabolism is a buildup of lactic acid, causing
acidosis, ie., occlusion of lower extremity artery

Shock

Oxygenated blood is not delivered throughout the body, causing anaerobic


metabolism and a buildup of lactic acid
Kidney illness
Decreased secretion or resorption of bicarbonate into the blood; decreased excretion
of H ions
Gastrointestinal (GI) illness: Lower GI contents are alkaline and diarrhea causes a loss of base solutions from the
diarrhea
body, resulting in acidic blood
Drugs: Diamox, Aldactone
Diamox causes loss of bicarbonate; Aldactone causes K retention and an increase
of serum K . The blood pushes the K into the cell to decrease serum K . This is a
normal compensatory mechanism. When K is pushed into the cell, H is pushed out
of the cell into the bloodstream, causing acidosis (remember, H is acid
Aspirin overdose
Acid is the end product of aspirin metabolism
Metabolic alkalosisuses Why
CAUSES
HOW THEY AFFECT ACID BASE BALANCE
Vomiting; bulimia; Removes stomach Removes stomach acid leaving the body alkaline
acid leaving the body alkaline
nasogastric (NG) tube suctioning
Excess antacid ingestion
Increases serum alkaline levels; kidneys may not be able to get rid of
excess
Blood transfusions
Preservative citrate is converted to bicarbonate (when blood is
administered, the client is getting bicarb too)
Sodium bicarbonate
IV administration in code situations may leave the
client too alkaline
Thiazide and diuretics
Loss of chlorine, which impedes manufacture of hydrochloric (HCL) acid,
making the body alkaline. Chlorine depletion enhances bicarbonate
resorption, increasing alkalinity
Baking soda
Home remedy for GI upset; very alkaline
Hypokalemia
Hypokalemia causes H to move into the cell, forcing K into the
bloodstream, increasing serum K . This is a normal compensatory
mechanism to correct the hypokalemia. This causes a decrease in
available hydrogen needed to make hydrochloric acid (HCL) which will

make the client more alkalotic (less acid makes more base)
Activation of rennin angiotensin system H ions secreted into the nephron add bicarbonate to the vascular space,
making the blood alkaline
Steroids
Sodium and water retention and K loss. Refer to hypokalemia to see how
this causes alkalosis
Dialysis
High bicarbonate dialysate is used to correct metabolic acidosis in endstage renal disease (ESRD). The dialysate is alkalinic
Licorice
Sodium and water retention and K loss. Refer to hypokalemia to see how
this causes alkalosis
70. Metabolic activities of the body require the precise regulation of acid-base balance, which is reflected in the pH
of extracellular fluids. Membrane excitability, enzyme systems, and chemical reactions depend on acid-base
balance being regulated within a narrow physiologic range to function in an optimal way. A clients ABG results
are pH 7.47, CO2 38, HCO3 29. The nurse should further assess:
A. Shock.
B. Hyperkalemia
C. Numbness and tingling of the extremities.
D. Increased pulse and respiratory rate.
ANSWER: C
RATIONALE: These ABGs are consistent with metabolic alkalosis. Numbness and tingling are associated with
metabolic alkalosis due to hypocalcemia.
OPTION A- Shock is associated with metabolic acidosis.
OPTION B- Hyperkalemia is typically associated with metabolic acidosis.
OPTION D- Increased pulse and respiratory rate are associated with respiratory alkalosis.
Metabolic alkalosis
SIGNS & SYMPTOMS
Arrhythmias, flattened T-wave
Decreased respirations, hypoventilation

CAUSES
Hypokalemia
Respiratory compensation to retain CO2.
Receptors in medulla of brain are depressed
due to excess of bicarbonate; eventually,
PCO2 will rise
Hypokalemia
Vomiting may have caused initial imbalance.
As K moves into the cells, serum K drops.
H moves into the bloodstream, increasing
serum acidity
Tightening of muscles, tetany, LOC changes, Hypocalcemia; alkalosis causes calcium to
seizures, tingling in fingers and toes
bind with albumin, making the calcium
inactive
LOC changes
The brain doesnt like it when pH is out of
balance; hypocalcemia
Hepatic encephalopathy
Alkalosis causes increased ammonia production
Metabolic acidosis
SIGNS & SYMPTOMS
Hyperkalemia

CAUSES
H builds up in the blood and the body compensates
by pushing the excess H ions into the cells (where they cant
be seen). When H moves into the cell, this disturbs K (whose

Arrhythmias
Increased respiratory rate

favorite place to live is in the cell alone), who moves out into the
bloodstream. This causes an increase in serum K
Bradycardia, peaked T-waves, prolonged PR interval, widened
QRS
Medulla in the brain is stimulated by excess H ions. Kussmaul
respirations compensate by blowing off CO2 (acid). Eventually,
PCO2 decreases
The brain does not like it when the pH is out of normal range
Hyperkalemia

Headache, decreased LOC, coma


Muscle twitching and burning, oral numbness,
weakness, flaccid paralysis (severe hyperkalemia)
Weakness, flaccid paralysis tingling and numbness Hyperkalemia and hypercalcemia
in the arms and legs

71. Acid-base imbalance is an abnormality of the human body's normal balance of acids and bases that causes the
plasma pH to deviate out of the normal range (7.35 to 7.45). Compensation for metabolic alkalosis occurs mainly
in the lungs, which retain carbon dioxide (CO2) through slower breathing, or hypoventilation (respiratory
compensation). Which of the following patients below is most likely manifesting metabolic alkalosis?
A. A 35 year old female with poor skin turgor, weak thread pulse, sunken eyes, and with hypokalemia
B. A 45 year old, male with BP 90/70, dry skin and oral mucosa, and with hyperkalemia
C. A 18 year old female with bounding pulse, distended neck and hand veins, and with hypokalemia
D. A 17 year old male with crackles upon lung auscultation, dyspneic when supine, ans with hyperkalemia
ANSWER: A
RATIONALE: Dehydration and hypokalemia are associated with metablic alkalosis and account for most of the
clinical manifestations seen in this type of acid-base imbalance.
In alkalosis, H+ move from the intracellular fluid into the extracellular fluid by diffusion. To keep the intracellular fluid
electro-chemically neutral, K+ move from the extracellular fluid into the intracellular fluid.)
Reference: Donna D. Ignatavicius& M. Linda Workman: Medical-Surgical Nursing: Patient-Centered Collaborative
Care, 6th Edition p.282
72. In COPD, less air flows in and out of the airways because of one or more of the following: the airways lose their
elasticity, the walls between air sacs are destroyed, the airways become inflamed, and there is clogging due to
increased mucus production. You are a nurse caring for a patient with chronic bronchitis, a COPD. Which of the
following ABG results would you most likely exhibit?
A. pH 7.46, HCO3- 20 meq/L
B. pH 7.30, HCO3- 19 meq/L
C. pH 7.31, HCO3- 26 meq/L
D. pH 7.47, HCO3- 28 meq/L
ANSWER: C
RATIONALE: Respiratory acidosis can result from decreased gas exchange and CO2 retention. Option a is
respiratory alkalosis. Option b is metabolic acidosis. Option d is metabolic alkalosis. All of which are partially
compensated
Reference: Donna D. Ignatavicius& M. Linda Workman: Medical-Surgical Nursing: Patient-Centered Collaborative
Care, 6th Edition p.282

73. The nurse needs to understand the physiology of fluid and electrolyte balance and acidbase balance to
anticipate, identify, and respond to possible imbalances in each. The nurse also must use effective
teaching and communication skills to help prevent and treat various fluid and electrolyte disturbances. A
client with a 4-day history of nausea and vomiting presents to the emergency department. The client is
hypoventilating and has a respiratory rate of 6 breaths/min. The electrocardiogram (ECG) monitor displays
tachycardia, with a heart rate of 120 beats/min. Arterial blood gases are drawn and the nurse reviews the results,
expecting to note which of the following?
A. A decreased pH and an increased CO2
B. An increased pH with an increased HCO3
C. An increased pH and a decreased CO2
D. A decreased pH and a decreased HCO3
ANSWER: B
RATIONALE: Clients experiencing nausea and vomiting would most likely present with metabolic alkalosis resulting
from loss of gastric acid, thus causing the pH and HCO3 to increase. Symptoms experienced by the client would
include hypoventilation and tachycardia.
Option A reflects a respiratory acidotic condition.
Option C reflects a respiratory alkalotic condition.
Option D reflects a metabolic acidotic condition
74. Many critical illnesses can upset a patient's acid-base balance, and a disturbance in acid-base equilibrium may
indicate other underlying diseases or organ damagePatient Flashs laboratory profile showed ABG results of pH
7.30, HCO3- 19 meq/L. Which of the following could not be a cause for these findings?
A. Salicylate intoxication
B. Diarrhea
C. Starvation
D. Airway obstruction
ANSWER: D
RATIONALE: Common causes of metabolic acidosis includes a) overproduction of hydrogen ions (hypermetabolic
state, excessive ingestion of acids e.g. salicylate, ethanol, methanol, inc. oxidation of fatty acids e.g. diabetic
ketoacidosis, starvation) b) underelimation of hydrogen ions (renal failure) c) underproduction of bicarbonate
(dehydration, renal/liver failure) d) overelimination of bicarbonate (diarrhea)
Option d is a common cause for respiratory acidosis.
Donna D. Ignatavicius& M. Linda Workman: Medical-Surgical Nursing: Patient-Centered Collaborative Care, 6th
Edition p.283
75. Accurately interpreting acid-base balance requires simultaneous measurements of arterial pH and plasma
electrolytes, as well as knowledge of compensatory physiologic mechanisms.During assessment, you noted
Patient Lich exhibiting polyuria and rapid-deep breathing. You know that these are his bodys compensatory
mechanism for which acid-base imbalance?
A. respiratory acidosis
B. respiratory alkalosis
C. metabolic acidosis
D. metabolic alkalosis
ANSWER: C
RATIONALE:

The respiratory system (rapid-deep breathing) compensates for metabolic acid-base imbalances. Increased H20 and
CO2 elimination during acidosis increases utilization of H+ thus decreasing its serum levels resulting to increase in
pH or blood alkalinity
Donna D. Ignatavicius& M. Linda Workman: Medical-Surgical Nursing: Patient-Centered Collaborative Care, 6th
Edition p.283
76. End-Stage Renal Disease (ESRD) is a progressive, irreversible deterioration in renal function which ultimately
leads to uremia or azotemia. Mrs. Chinita has been recently diagnosed with ESRD and has been told that she
will need to undergo dialysis soon to correct her fluid and electrolyte imbalances as well as metabolic waste
products. Which of the following clinical manifestations is not expected in Mirana?
A. crushing chest pain which radiates to the jaw and arm
B. BP: 167/99 mmHg
C. pink frothy sputum and dyspnea
D. breath smells like urine
Answer: D
RATIONALE: Pericarditis (due to irritation of the pericardial lining by uremic toxins) can be present in the patient,
which manifests as a sharp, pleuritic, retro-sternal (under the sternum) or left precordial (left chest) pain. A crushing
pain that radiates to the jaw and arm is characteristic of MI. Pulmonary edema (pink frothy sputum and dyspnea) is
caused by fluid overload. Hypertension is due to sodium and water retention or from activation of the RAAS. Uremic
fetor (breath has the odor of urine) is due to accumulation of waste products like urea.
Understanding Medical Surgical Nursing by Williams and Hopper, 3rd edition, 2007, p. 798-800; Brunner & Suddarths
Textbook of Medical-Surgical Nursing, 10th edition, 2003, p. 1326-1327
77. Changes in serum electrolytes and metabolites prove to be a major abnormality seen in ESRD. Which of the
following electrolyte values is least expected in Mrs. Chinita?
A. Hypocalcemia
B. Hyperphosphatemia
C. Hyperkalemia
D. Increased bicarbonate levels
ANSWER: D
RATIONALE:
Metabolic acidosis occurs because the kidneys are unable to excrete increased loads of acid. The kidney tubules are
unable to excrete ammonia (NH3-) and reabsorb sodium bicarbonate (HCO3-). This results in increased ammonia
levels (acid) and decreased bicarbonate (base) levels in the blood. Serum calcium and phosphate levels have a
reciprocal or inversely proportional relationship in the body. With decreased filtration through the glomerulus of the
kidney, there is an increase in the serum phosphate level and a reciprocal decrease in the serum calcium level.
Hyperkalemia is due to decreased renal excretion of potassium and metabolic acidosis (K moves out of cell).
Reference: p. 797-800, Understanding Medical Surgical Nursing by Williams and Hopper, 3rd edition, 2007; p. 13271328, Brunner & Suddarths Textbook of Medical-Surgical Nursing, 10th edition, 2003)
78. Potassium is the major intracellular electrolyte; in fact, 98% of the bodys potassium is inside the cells. The
remaining 2% is in the ECF, and it is this 2% that is important in neuromuscular function. Potassium influences
both skeletal and cardiac muscle activity. For example, alterations in its concentration change myocardial
irritability and rhythm. Under the influence of the sodium potassium pump and based on the bodys needs,
potassium is constantly moving in and out of cells. A client is being discharged from the hospital after being

treated for a decreased potassium level. In order for the client to maintain an appropriate potassium level, the
nurse suggests which food when providing discharge teaching?
A. Root beer
B. Dried beans
C. Butter
D. Ginger ale
ANSWER: B
RATIONALE:
Of the foods listed, dried beans are highest in potassium. Here is the list of foods rich in Potassium:
POTASSIUM RICH FOODS
FOODS WITH MINIMAL POTASSIUM CONTENT
Coffee
Butter
Cocoa
Margarine
Tea
cranberry juice or sauce
dried fruits
ginger ale,
dried beans
gumdrops or jellybeans,
wholegrain
hard candy
breads.
root beer
Bananas
sugar and honey.
Apricots
Baked potatoes (with skin)
Fresh meat
Lentils
Avocado
Cantaloupe
OPTION B & D- Peas and Nuts are excellent source of B complex vitamins
OPTION C- Fowls are any other domesticated bird kept for its eggs or flesh, e.g., the turkey, duck, goose.
All fowl is rich in protein, with as much of the nutrient as red meat.
Reference: Understanding Medical-Surgical Nursing , Second Edition, by Linda S. Williams, 2003 by F. A. Davis
Company
79. Nurses need to carefully plan nursing care activities to ensure safe administration of medications. Clinical
Instructor Nerry is supervising her student, Ejay in their clinical duty in the medical ward. She assigned Ejay to
administer potassium chloride intravenously as prescribed to a client with hypokalemia. Nurse Nerry determines
that Ejay is not unprepared for this procedure if he states that which of the following is part of the plan for
preparation and administration of the potassium aside from?
a. Obtaining a controlled IV infusion pump
b. Monitoring urine output during administration
c. Diluting in appropriate amount of normal saline
d. Preparing the medication for bolus administration

RATIONALE:
OPTION D- Potassium chloride is never given by bolus (IV push). Giving potassium chloride by IV push can result in
cardiac arrest. OPTION A- Potassium chloride administered intravenously must always be diluted in IV fluid and
infused via a pump or controller. The usual concentration of IV potassium chloride is 20 to 40 mEq/L.
OPTION B- . The nurse monitors urinary output during administration and contacts the physician if the urinary output
is less than 30 mL/hr.
OPTION C- Dilution in normal saline is recommended, but dextrose solution is avoided because this type of solution
increases intracellular potassium shifting. The IV bag containing the potassium chloride is always gently agitated
before hanging. The IV site is monitored closely because potassium chloride is irritating to the veins and the risk of
phlebitis exists

REFERENCE: Understanding Medical-Surgical Nursing , Second Edition, by Linda S. Williams, 2003 by F. A. Davis
Company
80. Fluid overload, sometimes called overhydration, is a condition in which a patient has too much fluid in
the body. Most of the problems related to fluid overload result from too much fluid in the bloodstream or
from dilution of electrolytes and red blood cells. Mr. Chinito is a 32-year-old man with a congenital heart
problem. He has been recovering from congestive heart failure with a nursing diagnosis of fluid volume excess
(FVE). A nurse examining the client would expect to find:
A. Moist mucous membranes.
B. Postural hypotension.
C. Tea colored urine
D. Weak, rapid pulse.
ANSWER: A
RATIONALE: Moist mucous membranes are the only sign and symptom listed consistent with fluid volume excess.
The vital sign changes seen in the patient with fluid overload are the opposite of those found in patients with
dehydration. The blood pressure is elevated, pulse is bounding, and respirations are increased and shallow. The
neck veins may become distended, and pitting edema in the feet and legs may be present. The skin is pale and cool.
The kidneys increase urine output, and the urine appears diluted, almost like water. The patient rapidly gains weight.
In severe fluid overload the patient develops moist crackles in the lungs, dyspnea, and ascites (excess peritoneal
fluid).
Reference: Understanding Medical-Surgical Nursing , Second Edition, by Linda S. Williams, 2003 by F. A. Davis
Company
81. Overhydration may be as serious as dehydration. It generally occurs in children who are receiving IV fluid. The
excess fluid in these instances is usually extracellular. The condition is serious because the ECF overload can
lead to cardiovascular overload and cardiac failure. The nurse is caring for an infant receiving intravenous fluid.
Signs of fluid overload in an infant include:
A. Swelling of the hands and increased temperature
B. Swelling of the feet and increased temperature
C. Decreased heart rate and decreased blood pressure
D. Increased heart rate and increased blood pressure
ANSWER: D
RATIONALE: Signs of fluid overload in an infant include increased heart rate and increased blood pressure.
Temperature would not be increased by fluid overload; therefore, answers A and C are incorrect. Heart rate and
blood pressure are not decreased by fluid overload; therefore, answer D is incorrect
Overloading the circulatory system with excessive IV fluids causes increased blood pressure and central
venous pressure. Signs and symptoms of fluid overload include moist crackles on auscultation of the lungs, edema,
weight gain, dyspnea, and respirations that are shallow and have an increased rate. Possible causes include rapid
infusion of an IV solution or hepatic, cardiac, or renal disease. The risk for fluid overload and subsequent pulmonary
edema is especially increased in elderly patients with cardiac disease; this is referred to as circulatory overload. The
treatment for circulatory overload is decreasing the IV rate, monitoring vital signs frequently, assessing breath
sounds, and placing the patient in a high Fowlers position. The physician is contacted immediately. This complication
can be avoided by using an infusion pump for infusions and by carefully monitoring all infusions. Complications of
circulatory overload include heart failure and pulmonary edema.
82. The Association of Nursing Service Administrators of the Philippines (ANSAP) is the one responsible for making
the guidelines for the IV Therapy Trainings in the Philippines. They also conduct these trainings in hospitals or
training centers in different parts of the Philippines. The nurse is caring for a client with a peripheral IV. Which of
the following would indicate that the client is experiencing hypervolemia?

I.
II.
III.
IV.
V.
VI.
A.
B.
C.
D.

nausea and vomiting


headache
dyspnea
hypertension
fever
tachycardia

iv and vi
iii, iv, and vi
i, iv, and vi
I and iii

ANSWER: B
RATIONALE: Clinical manifestations of hypervolemia with intravenous therapy include dyspnea, hypertension,
tachycardia, coughing, pulmonary edema, cyanosis, rales, and increased venous pressure. Nausea, vomiting, and
fever are clinical manifestations of a pyrogenic reaction.
Reference: Brunner 2008, p. 313
83. Hyperkalemia is defined as a serum potassium concentration greater than approximately 3.5-5.5 mEq/L in
adults. Hyperkalemia is more likely to happen in which of the following conditions?
A. Taking of corticosteroids
B. Prolonged gastric suctioning
C. Crush injury
D. Alkalosis
ANSWER: C
RATIONALE: A crush injury occurs when a body part is subjected to a high degree of force or pressure, usually after
being squeezed between two heavy objects.Hyperkalemia can be caused by muscle breakdown due to a traumatic
crush injury. Potassium is released from the injured cells and hyperkalemia results. Taking of corticosteroids may
result in hypernatremia and hypokalemia. Alkalosis and prolonged gastric suctioning may also cause hypokalemia.
84. Many cardiac surgical procedures are possible because of cardiopulmonary bypass. The procedure
mechanically circulates and oxygenates blood for the body while bypassing the heart and lungs. EJ is scheduled
for CBP surgery. A possible electrolyte imbalance secondary to this invasive surgery is hyperkalemia. Nurse
Franco will suspect the client to exhibit the following, except:
A. Muscle weakness
B. Palpitations
C. Paresthesias
D. Elevated U wave
ANSWER: D
RATIONALE: An elevated U wave is specific to hypokalemia. Paresthesia is seen in hyperkalemia. Muscle
weakness and palpitations are seen in both hyerkalemia and hypokalemia.
Hyperkalemia is defined as a serum potassium Hypokalemia is generally defined as a serum potassium
concentration greater than approximately 3.5-5.5 mEq/L level of less than 3.5 mEq/L (3.5 mmol/L).
in adults
S/Sx:
S/Sx:
asymptomatic.
Asymptomatic
Weakness and fatigue are the most common
Weakness and fatigue (most common)

complaints

Frank muscle paralysis

Dyspnea

Palpitations

Chest pain
Nausea or vomiting
Paresthesias
ECG:

Early ECG changes of hyperkalemia, typically seen

at a serum potassium level of 5.5-6.5 mEq/L, include

the following:

Tall, peaked T waves with a narrow base, best

seen in precordial leads[4]

Shortened QT interval
ST-segment depression
At a serum potassium level of 6.5-8.0 mEq/L, the

ECG typically shows the following:


Peaked T waves
Prolonged PR interval
Mgt:
Decreased or disappearing P wave
1.
Widening of the QRS

Amplified R wave

At a serum potassium level higher than 8.0 mEq/L,


the ECG shows the following:
Absence of P wave
2.
Progressive QRS widening

Intraventricular/fascicular/bundle branch blocks


The progressively widened QRS eventually
3.
merges with the T wave, forming a sine wave

pattern. Ventricular fibrillation or asystole


follows.
4.
Mgt: Severe
1. IV Calcium to ameliorate cardiac toxicity
2. IV glucose and insulin infusion to enhance
potassium uptake by cells
3. Correct severe metabolic acidosis with sodium
bicarbonate
4. Emergency dialysis

Muscle cramps and pain (severe cases)


Worsening diabetes control or polyuria
Palpitations
Psychological symptoms (eg, psychosis,
delirium, hallucinations, depression)
Ventricular dysrhythmia
Prolongation of QT interval
ST-segment depression
T-wave flattening
Appearance of U waves
Ventricular arrhythmias (eg, premature
ventricular contractions [PVCs], torsade de
pointes, ventricular fibrillation)[40]
Atrial arrhythmias (eg, premature atrial
contractions [PACs], atrial fibrillation)
Reduction of potassium losses
Discontinue diuretics/laxatives
Use potassium-sparing diuretics if diuretic
therapy is required
Replenishment of potassium stores
potassium chloride
Evaluation for potential toxicities
Generally, cardiac in nature
Determination of the cause to prevent future
episodes, if possible

85. Cancer and its treatment may lead to a range of potentially life-threatening conditions that require urgent action
to correct them. One of the oncologic emergencies that merit urgent attention is:
A. Hypercalcemia
B. Hypernatremia
C. Hyponatremia
D. Hypocalcemia
ANSWER: A
RATIONALE: Abnormalities in calcium levels may be sufficiently severe to constitute metabolic emergencies.
Hypercalcemia is encountered more frequently than hypocalcemia. Although the incidence of hypercalcemia has not
been estimated accurately, hypercalcemia is notably prevalent in adults with cancer.

This is the most common serious metabolic disorder associated with malignancy, affecting up to one third of cancer
patients at some point in their disease course.[2]Malignancies most commonly associated include lung cancer, breast
cancer, renal cancer, multiple myeloma and adult T-cell lymphoma. Its symptoms may mimic the features of terminal
malignancy. Hypercalcaemia is a poor prognostic indicator in malignant disease and may indicate uncontrolled
tumour progression and metastasis. The 30-day mortality rate of cancer patients admitted to hospital with
hypercalcaemia is almost 50%.
86. Posttraumatic stress disorder (PTSD) is a disturbing pattern of behavior demonstrated by someone who has
experienced a traumatic event. All of the following are significant factors that increase the likelihood of the
occurrence of PTSD, except:
A. Proximity of event
B. Duration of trauma
C. Severity of effect
D. Stress level of the person
ANSWER: A
RATIONALE: The severity and duration of the trauma and the proximity of the person to the event are the most
important factors affecting the likelihood of developing PTSD
Reference: APA, 2000, quoted from Videbeck, 2nd ed, page 224
87. In most cases of PTSD, the person was exposed to an event that posed a threat of death or serious injury, to
which he responded with intense fear, helplessness, or terror. Edward was a survivor of the 9/11 bombing of the
World Trade Center. Every time he hears a sudden loud sound, he is frozen and covers his ears in terror. He
has a difficult time sleeping at night, and is constantly terrified of airplanes. Under which cluster of symptoms do
these fall into?
A. Reliving the event
B. Avoiding reminders of the event
C. Traumatic stage
D. Hyperarousal
ANSWER: C
RATIONALE: In PTSD, there are three clusters of symptoms present: reliving the event; avoiding reminders of the
event; and being on guard, or hyperarousal. The victim feels a numbing of general responsiveness and shows
persistent signs of increased arousal such as insomnia, hyperarousal or hypervigilance, irritability, or angry outbursts
Re-Experiencing Symptoms
Frequently having upsetting thoughts or memories about a traumatic event.
Having recurrent nightmares.
Acting or feeling as though the traumatic event were happening again, sometimes called a "flashback."
Having strong feelings of distress when reminded of the traumatic event.
Being physically responsive, such as experiencing a surge in your heart rate or sweating, to reminders of
the traumatic event.
Avoidance Symptoms
Making an effort to avoid thoughts, feelings, or conversations about the traumatic event.
Making an effort to avoid places or people that remind you of the traumatic event.
Having a difficult time remembering important parts of the traumatic event.
A loss of interest in important, once positive, activities.
Feeling distant from others.
Experiencing difficulties having positive feelings, such as happiness or love.
Feeling as though your life may be cut short.
Hyperarousal Symptoms
Having a difficult time falling or staying asleep.

Feeling more irritable or having outbursts of anger.


Having difficulty concentrating.
Feeling constantly "on guard" or like danger is lurking around every corner.
Being "jumpy" or easily startled.
Reference: Videbeck, 2nd Ed, page 231; http://ptsd.about.com/od/symptomsanddiagnosis/a/PTSDsymptoms.htm
88. Nursing care for persons suffering from PTSD follows the same format of ADPIE. Yeng is the nurse caring for
Enchong. He suddenly screams in terror, and appears to be reliving the 9/11 incident. Which of the following is
Nurse Yengs priority intervention?
A. Reorient the patient to reality to lessen the anxiety felt by the patient.
B. Ensure that the client will not harm himself or others.
C. Perform deep breathing exercises with the patient to promote relaxation.
D. Ask the patient to verbalize what he is seeing so that she can help him through it.
ANSWER: B
RATIONALE: The clients safety is a priority, especially in acute attacks. The nurse continually must assess the
clients potential for self-harm or suicide and take action accordingly.
Reference: Videbeck, 2nd Ed, page 235
89. PTSD is classified under Anxiety Disorders. It is similar to other disorders, such as the Acute Stress Disorder
(ASD). How does the DSM distinguish the two diagnoses?
A. In PTSD, the triggering event is proximal to the person affected, which is the reason for the recurrence and
long-term effects of the symptoms. In ASD, the triggering event may or may not be personally related to the
person, but it depends on how the person copes.
B. In PTSD, the symptoms should occur more than 3 months after a triggering event. In ASD, symptoms
should appear within the first 4 weeks after the event.
C. In PTSD, the symptoms are more severe, and usually involve nightmares and insomnia. In ASD, the
symptoms have a much shorter duration of effect.
D. In PTSD, the person is able to identify the specific triggering event. In ASD, the triggering event may be
general, and the person may not be able to identify which event it was exactly.
ANSWER: B
RATIONALE: The primary difference between ASD and PTSD is the duration of the symptoms nd the formers
emphasis on dissociatie reaction to the trauma. ASD refers to symptoms manifested during the period form 2 days o
4 weks post trauma, whereas PTSD can only be diagnosed from 4 weeks. In terms of dissociation, the diagnosis of
ASD requires that the individual has at least 3 of the ff: 1. A subjective sense of numbing or detachment, 2. Reduced
awareness of ones surrounding, 3. Derealization, 4. Depersonalization, 5. Dissociative amnesia. Whereas PTSD
requires 3 avoidance or numbing symptoms and 2 arousal symptoms, the ASD criteria require marked avoidance
and arousal (http://focus.psychiatryonline.org/data/Journals/FOCUS/4332/foc00311000335.pdf)
In PTSD, the symptoms occur 3 months or more after the trauma, which distinguishes PTSD from acute stress
disorder. This DSM-IV-TR diagnosis is appropriate when symptoms appear within the first month after the trauma
and do not persist longer than 4 weeks (Videbeck, 2nd Ed, page 221)
90. Posttraumatic stress disorder (PTSD) is a disturbing pattern of behavior demonstrated by someone who
has experienced a traumatic eventfor example, a natural disaster, combat, or an assault. The person
persistently re-experiences the trauma through memories, dreams, flashbacks, or reactions to external
cues about the event and, therefore, avoids stimuli associated with the trauma. Mendiolaz, a Yolanda
survivor is diagnosed with PTSD. He says to the nurse, I hate God, he is the reason why my family died From
this statement, the nurse assesses which of the following in John?
A. Repressed anger

B. Survivors guilt
C. Intrusive thoughts
D. Spiritual distress
ANSWER: D
RATIONALE: Spiritual Distress involves disruption in the life principle that pervades a persons entire being. It may
be related to inability to practice spiritual ritual, conflicts between spiritual beliefs and other aspects of life, or the
crisis of illness, suffering or death.
OPTION B- Survivors Guilt is a expression of normal concern for other human beings singly and as a society which
appears after a disaster or trauma. The person who feels the guilt has not been affected or hit by the trauma but
there was a fair probability that he or she could have been the victim. The characteristic symptoms of Survivors Guilt
includes primarily, Guilt and self-blame, anxiety, depression, sleep disturbances, emotional liability, loss of drive ,
lower motivation and morale and sometimes, physical complaints.
OPTION A-Repressed Anger is a anger which is made is placed on the unconscious state of mind as a psychological
defense mechanism because the emotion is unacceptable.
OPTION C- Recurring, Intrusive Thoughts in PTSD almost always cause a strong urge of emotion- anger, fear,
humiliation, helplessness. When youre asleep, they can show up as nightmares. In the day time, they pull you away
from the present time, sometimes making you behave in some ways that dont make sense to people around you.
The best known type of these intrusions are flashbacks.
Reference: What Nurses KnowPTSD by Mary E. Muscari, Demos Medical Publishing, 2012
91. Spouse or partner abuse is the mistreatment or misuse of one person by another in the context of an intimate
relationship. Most abused women do not seek direct help for the problem. Because this issue is delicate and
sensitive and many abused women are afraid or embarrassed to admit the problem, nurses must be skilled in
asking appropriate questions about abuse. All of the following questions can be asked to establish relationship
and develop empathy with the patient, except:
A. Do you feel safe in your relationships?
B. Do you think that your partner is abusing you?
C. Are there situations in your relationships where you have felt afraid?
D. People in relationships/marriages often fight; what happens when you and your partner disagree?
ANSWER: B
RATIONALE: A possible framework of questions to ask is by using the acronym SAFE (Stress/Safety, Afraid
/Abused, Friends/Family, and Emergency plan). The first two categories are designed to detect abuse. The nurse
should ask questions in the other two categories if abuse is present (Videbeck, 2nd Ed, page 222).
Particularly if the abuse has been happening over a long period of time, the victim is likely to feel depressed, insecure
and lacking in confidence and self-esteem. She may be extremely afraid of the situation, and that fear may include a
fear of talking to anyone about what has been taking place. Women who experience domestic violence often try to
explain it to themselves, and others, by seeing it as their responsibility or fault, and the response of others to their
situation may have reinforced this view. Before asking direct questions, it may help to begin with some indirect ones
to help in establishing a relationship with the patient and developing empathy, for example:
Is everything alright at home?
Are you being looked after properly/is your partner taking care of you?
Do you get along well with your partner?
Women may not disclose violence unless asked directly. Direct questions may be asked after establishing
relationship with the patient.

The term "domestic violence", also referred to as "spouse/partner abuse", is used to describe a form of family
violence. Domestic violence includes physical abuse, sexual violence, psychological and/or emotional abuse of a
woman by her mate or companion.
Why Is Domestic Violence So Common?
a lack of understanding that verbal and physical violence are learned behaviors, often learned from role models
such as parents, relatives or friends;
a lack of understanding that violence in relationships is often used as a way to reduce emotional stress, as a
defense mechanism, or as a way to maintain control in the relationship;
a lack of recognition that there is a high correlation between alcohol and substance abuse and domestic
violence;
continual exposure to violent behavior in entertainment, sports and the media;
reinforcement of sexual roles condoning aggressive and violent behavior by males; and,
a lack of public awareness regarding the severity of the problem with many still believing it is a private matter
within the family instead of a criminal issue and, therefore, best left alone.
What About the Victim? Although there is no one profile of the battered woman, there are some common
characteristics which victims of spouse/partner abuse share.
come from all sectors of society, from every social, economic, religious, and racial group, from all walks of life
and all lifestyles.
often feel degraded and worthless.
A battered womans lack of positive self-esteem may keep her from telling anyone about her abuse. She may
fear being seen as a "failure" as a wife and mother, or worse yet, she may believe she "deserves"" the
mistreatment.
may have been brought up to believe that it is her duty to keep the family together, no matter what the cost. She
will, therefore, endure the abuse for the sake of the children, often leaving when the violence is directed at them.
may not reveal her abuse because she believes that society generally ignores domestic violence, or that she will
be blamed for provoking or accepting the violence.
frequently totally dependent on her husband financially and often faces severe economic hardship if she leaves.
If the victim married young and has several children, she may have few job skills to make her feel confident
enough to support her family without the financial support of a spouse or partner.
often forced into isolation by the battering spouse or partner, cut off from family and friends. This isolation further
victimizes her by giving her abuser power and control over her life.
What About the Men Who Batter? There is no "typical profile" for men who batter. However, a male batterer
may exhibit one or more of the following characteristics:
often denies the existence of violence and its effect on the victim and other members of the family;
often exhibits extreme possessiveness and jealousy, thus isolating the victim from family and friends;
often refuses to accept responsibility for the abuse, blaming his behavior on stress, alcohol, drugs or the victim;
may have grown up in a home where male dominance over women was modeled physically or verbally; and,
may believe in a traditional male sex role, or have developed a strong negative attitude toward women.
The following are some of the reasons given for why men batter.
Battering is a learned behavior, not a mental disorder.
Many batterers were raised in a violent home where they witnessed the abuse of their mother, siblings, or
perhaps were themselves a victim of childhood abuse.
Witnessing domestic violence in the childhood home is the most common risk factor for becoming a batterer in
adulthood.
The batterer has learned to used physical force as a way to maintain power and control in his relationships with
women. Battering is the ultimate expression of a belief in male dominance over females.
The batterer has learned to use physical violence as a means to handle anger, frustration, or guilt, and lacks the
communication skills necessary to handle these emotions in non-violent ways.
The batterer generally has low self-esteem and low self-control, often displacing his anger at his boss, or himself
onto his spouse/partner and children.

A batterer may experience some remorse after the battering and even seek forgiveness from his victim,
promising it will never happen again. Such promises are rarely kept. Good intentions will not cure battering.
Men who batter choose to do so and, until recently, there has been no consequence for this behavior.
Acts of violence committed within the family, which would be considered assaults with penalties if perpetrated on
a stranger, have gone unnoticed by society and unpunished

92. Ineffective Coping is the inability to form a valid appraisal of the stressors, inadequate choices of practiced
responses, and/or inability to use available resources. For a client who has experienced abuse as a child, all of
the following are immediate objectives, except
A. The client will express feelings of helplessness, fear, anger, guilt, anxiety, etc
B. The client will identify support systems outside the hospital
C. The client will demonstrate decreased withdrawn, depressive, or anxious behaviors
D. The client will demonstrate a decrease in stress-related symptoms
ANSWER: B
RATIONALE: This is an objective once the acute stage has been stabilized. All the other choices are immediate
objective (Videbeck, 2nd Ed, page 237)
Tips for talking to an abused child
Avoid denial and remain calm. A common reaction to news as unpleasant and shocking as child abuse is
denial. However, if you display denial to a child, or show shock or disgust at what they are saying, the child may
be afraid to continue and will shut down. As hard as it may be, remain as calm and reassuring as you can.
Dont interrogate. Let the child explain to you in his or her own words what happened, but dont interrogate the
child or ask leading questions. This may confuse and fluster the child and make it harder for them to continue
their story.

Reassure the child that they did nothing wrong. It takes a lot for a child to come forward about abuse.
Reassure him or her that you take what is said seriously, and that it is not the childs fault.
Safety comes first. If you feel that your safety or the safety of the child would be threatened if you try to
intervene, leave it to the professionals. You may be able to provide more support later after the initial
professional intervention.

93. Child abuse may be physical (the child is beaten or burned), or it may be neglect (the child is not fed, clothed or
supervised properly). Abuse may also be psychological or emotional. Allan is a 4 year old child you see in the
Emergency Room. Her mother tells you Allan fell off a swing in the backyard. Allan has a broken arm, a broken
rib, and multiple bruises on his chest and back. You also noticed in his chart that he was seen in the same
emergency room a month ago for a burn on the palm of his hand. When you mentioned to his mother that Allans
injuries seem extreme for a simple fall, his mother says, Allan isnt a good boy. I guess hes also clumsy.Which
finding in Allans background is a typical finding of child abuse?
A. Allans father smokes a pack of cigarette per day.
B. Allans mother describes her as not good.
C. Allans mother works part time for low-pay.
D. Allans birthday is in the fall of the year.
ANSWER: B
RATIONALE: Many of the parents who abuse have high expectations of a child but the child failed to meet it. It is
manifested when a parent is overly concerned about the physical appearance or sex of the child (She should have
been a boy or Shes not good, She looks like her bastard father). These statements imply that a parent has
difficulty accepting the child and is a typical reason for several cases of child abuse. Birthday is not related to child
abuse. Although low-income family is one of the risk factors of child abuse, it is not specific to the situation presented
above as it was not mentioned. Excessive parental use of alcohol, not smoking, is strongly associated with child
abuse.
Reference: Christian, 2003
94. Nemia, a nursing instructor, teaches a group of nursing students about violence in the family. Which statement
by a student indicates need for further teaching?
A. Abusers usually have poor self-esteem.
B. Abusers use fear and intimidation
C. Abusers often are jealous or self-centered.
D. Battering is a mental disorder.
ANSWER: D
RATIONALE: Personal characteristics of abusers include low self-esteem, immaturity, dependence, insecurity, and
jealousy. Abusers often will use fear and intimidation to the point at which their victims will do anything just to avoid
further abuse. Battering is a learned behavior, and is not a mental disorder, often learned from role models such as
parents, relatives or friends. Many batterers were raised in a violent home where they witnessed the abuse of their
mother, siblings, or perhaps were themselves a victim of childhood abuse. Witnessing domestic violence in the
childhood home is the most common risk factor for becoming a batterer in adulthood.
95. Rena, a 10-year-old girl who has been referred for evaluation for drawing sexually explicit scenes in her
textbooks says to the psychiatric nurse, I just feel like it. Which response is therapeutic for the nurse to make in
order to assess abuse-related symptoms?
A. I am concerned about you. Are you being abused now or have you ever been abused?
B. Well, a picture paints a thousand words.
C. You just felt like destroying your text books?

D. Your parents and teachers are very concerned about your drawings.
ANSWER: A
RATIONALE: The behaviors that this child engaged in are a warning signal of distress. Option A is the only option
that specifically addresses abuse. In option b, the nurse is insensitive, sarcastic, and intrusive. In option c, the nurse
is assessing the clients destructive behaviors, not the possible sexual abuse history. In option d, although the nurse
is trying to assess the clients abuse-related symptoms, the nurse uses indirect means rather than straightforward
expressions of the nurses concern. Use the process of elimination and therapeutic communication techniques,
focusing on the subject of the question. Noting the strategic words assess abuse-related symptoms will direct you to
option a.
A variety of techniques can be used in trying to elicit information from the child. The focus here is on
techniques most useful with young children. Appropriate questions and several types of media or props
anatomically explicit dolls, anatomical drawings, picture drawing, storytelling, and the doll house are discussed. The
interviewer should assume that the more open-ended the question, the greater confidence he/she should have in the
child's responses.
General questions are frequently used as opening questions when an adult comes in for assessment or
treatment. For example, if an adult rape victim comes to a mental health professional, the therapist might begin by
asking, "Tell me why you came to see me today." This question is likely to elicit an account of the rape.
Unfortunately, these general questions are less useful with young children. Typical responses from them are, "No," or
"I don't remember" (despite the care the accompanying adult might have taken in preparing the child). Alternatively
young children may acknowledge that they know why they are being interviewed but say they don't want to talk about
it. The children may also give vague responses such as "to talk about the bad things" or "to say what Grandpa did."
However, they may fail or refuse to elaborate. More directive questions are needed.
Reference: https://www.childwelfare.gov/pubs/usermanuals/sexabuse/sexabusee.cfm
96. During group session for battered women, a client says, I was abused by my father and then my husband, so I
finally stabbed my husband when he came after me, but no one believed me because my husband, the big
shot, can lie to anyone and be believed. If no one in the group responds, which of the following is the
therapeutic response by the nurse?
A. Seems as if you went from one abusing man to another. How come you were able to stab your own
husband?
B. Im glad that finally you were able to defend yourself. I think we should allow others to share their stories
too.
C. Yes, everyone here was ill-used and abused. Youll do fine
D. That is a horrible experience for you to undergo
ANSWER: D
RATIONALE: The therapeutic response is one that uses reflection and facilitates the clients feelings .
OPTION A: The first statement is correct which applies the restating/paraphrasing technique. However, the second
statement made the option wrong. Questioning is not therapeutic.
OPTION B: Both statements are not therapeutic the first statement gives approval while the second statement
introduces unrelated topics.
OPTION C: Reassuring is not therapeutic.

Reference:
http://www.rhrc.org/resources/gbv/comm_manual/day3.pdf
97. Rape is sexual activity such as intercourse or penetration of a body orifice by a penis or other object under
actual or threatened force. Nurses working in emergency departments may be asked to testify in court about the
victims appearance after the incident, although the documentation in the chart is usually all that is necessary.
Any clothing that is ripped or stained should be considered as evidence of violent assault and secured according
to hospital policy. When gathering evidence from a victim of rape, the nurse should place the victims clothing in
a:
A. Plastic zip-lock bag
B. Rubber tote
C. Paper bag
D. Padded manila envelope
ANSWER: C
RATIONALE: PAPER BAG . A paper bag should be used for the victims clothing because it will allow the clothes to
dry without destroying evidence. The basics of care management for patients with traumatic injury include an

understanding that trauma in any patient (living or dead) has potential legal, or forensic, implications. Hence, proper
management from both a medical and forensic perspective is essential.
The rational for collecting forensic evidence is to link a suspect to the in order to collect suitable forensic
evidence. The health worker must understand the types of evidence that may be present in sexual assault cases.
Wet or blood-stained clothing should be air dried (do not use extreme sources of heat e.g., a hairdryer);
once dried, place in a paper (not plastic) bags; do not store wet clothing; ensure packaging is properly labeled and
delivered to laboratory asap; if delivery to the laboratory is delayed, store in a cool dry place that is secure. The
clothing will be tested for biological stains such as blood, semen, and saliva from the suspect; hair from the assailant;
foreign materials such as grass, sol fibres or debris from the suspect or the crime scene; tears, loss of buttons and
other damage contain as a result of the abuse.
Valuables should be placed in the hospital safe or clearly documented as to which family member they were
given. If a police officer is present to collect clothing or any other items from the patient, each item is labeled. The
transfer of custody to the officer, the officers name, the date, and the time are documented.
Similarly, samples containing biological evidentiary material such as DNA should be stored in a cool dry
environment or refrigerated to prevent putrefaction (decomposition) which may render the sample unsuitable for
analysis. Care must be taken to avoid contamination of the sample by the health worker who may inadvertently add
material such as hair, blood or bacteria during the collection, preservation, or handling of the sample
OPTIONS A and B - are incorrect because plastic and rubber retain moisture that can deteriorate evidence.
OPTION D - is incorrect because padded envelopes are plastic lined, and plastic retains moisture that can
deteriorate evidence.
In the immediate aftermath of a sexual assault, the most important thing is for the victim to get to a safe
place. Whether it be the victims home, a friends home or with a family member, immediate safety is what matters
most. When a feeling of safety has been achieved, it is vital for the victim to receive medical attention, and strongly
recommended for the victim to receive a forensic examination.
Preserving DNA evidence can be a key to identifying the perpetrator in a sexual assault case, especially
those in which the offender is a stranger. DNA evidence is an integral part of a law enforcement investigation that can
build a strong case to show that a sexual assault occurred and to show that the defendant is the source of biological
material left on the victims body.
VICTIMS SHOULD MAKE EVERY EFFORT TO SAVE ANYTHING THAT MIGHT CONTAIN THE PERPETRATORS DNA,
THEREFORE A VICTIM SHOULD NOT

Bathe or shower
Use the restroom
Change clothes
Comb hair
Clean up the crime scene
Move anything the offender may have touched
Even if the victim has not yet decided to report the crime, receiving a forensic medical exam and keeping the
evidence safe from damage will improve the chances that the police can access and test the stored evidence at a
later date.
Reference: Brunner and Suddharts Medical Surgical Nursing;
http://www.dundee.ac.uk/forensicmedicine/Crimes_against_Women_and_Children/D11-SexualOffences.pdf
http://www.rainn.org/get-information/aftermath-of-sexual-assault/preserving-and-collecting-forensic-evidence
98. Somatoform disorders are characterized by physical symptoms suggesting medical disease, but without
demonstrable organic pathology or known pathophysiological mechanism to account for them. They are
classified as mental disorders because pathophysiological processes are not demonstrable or understandable
by means of existing laboratory procedures, and there is either evidence or strong presumption that
psychological factors are the major cause of the symptoms. Kimmy, your co-staff nurse with chronic low back
pain receives assistance for her tasks from your other colleagues. This is best described as:

A.
B.
C.
D.

Primary Gain
Attention seeking
Secondary gain
Malingering

ANSWER: B
RATIONALE: Somatization is defined as the transference of mental experiences and states into bodily symptoms.
Somatoform disorders can be characterized as the presence of physical symptoms that suggest a medical
condition without a demonstrable organic basis to account fully for them. The three central features of somatoform
disorders are as follows:
Physical complaints suggest major medical illness but have no demonstrable organic basis.
Psychological factors and conflicts seem important in initiating, exacerbating, and maintaining the
symptoms.
Symptoms or magnified health concerns are not under the clients conscious control
OPTION C-Secondary Gains are unintentionally sough benefits that results from illness, such as support that
otherwise might not be available. These benefits serve to reinforce illness behavior.
OPTION A- Primary gains are symbolic resolutions of unconscious conflict that decrease anxiety and keep the
conflict from awareness.
OPTION B & D- Attention seeking and malingering are deliberate behaviors. Malingering is the intentional
production of false or grossly exaggerated physical or psychological symptoms; it is motivated by external incentives
such as avoiding work, evading criminal prosecution, obtaining financial compensation, or obtaining drugs. People
who malinger have no real physical symptoms or grossly exaggerate relatively minor symptoms. Their purpose is
some external incentive or outcome that they view as important and results directly from the illness. People who
malinger can stop the physical symptoms as soon as they have gained what they wanted.
Reference: Guggenheim, F. Somatoform disorders. In Sadock, B.J. & Sadock, A. V., eds. Comprehensive Textbook
of Psychiatry. Philadelphia, PA: J. B. ..(2000)
99. Dissociation is a subconscious defense mechanism that helps a person protect his or her emotional self from
recognizing the full effects of some horrific or traumatic event by allowing the mind to forget or remove itself from
the painful situation or memory. Dissociation can occur both during and after the event. Rhodora has a history of
childhood physical and sexual abuse. She was diagnosed with Dissociative Identity Disorder (DID) 6 years ago.
She has been admitted to the psychiatric unit following a suicide attempt. The primary nursing diagnosis for
Rhodora would be:
A. disturbed personal identity related to childhood abuse.
B. disturbed sensory perception related to repressed anxiety.
C. risk for suicide related to unresolved grief.
D. disturbed thought processes related to memory deficit.
ANSWER: C
RATIONALE: The clients safety is a priority. The nurse continually must assess the clients potential for self-harm or
suicide and take action accordingly. The nurse and treatment team must provide safety measures when the client
cannot do so. To increase the clients sense of personal control, he or she must begin to manage safety needs as
soon as possible. The nurse can talk with the client about the difference between having self-harm thoughts and
taking action on those thoughts: having the thoughts does not mean the client must act on those thoughts. Gradually
the nurse can help the client to find ways to tolerate the thoughts until they diminish in intensity. The nurse can help
the client learn to go to a safe place during destructive thoughts and impulses so that he or she can calm down and
wait until they pass. Initially this may mean just sitting with the nurse or around others. Later the client can find a safe
place at home, often a closet or small room, where he or she feels safe. The client may want to keep a blanket or
pillows there for comfort and pictures or a tape recording to serve as reminders of the present.

Dissociative identity disorder (DID) formerly called multiple personality disorder, is characterized by the
existence of two or more personalities in a single individual. Only one of the personalities is evident at any given
moment, and one of them is dominant most of the time over the course of the disorder. Each personality is unique
and composed of a complex set of memories, behavior patterns, and social relationships that surface during the
dominant interval. The transition from one personality to another is usually sudden, often dramatic, and usually
precipitated by stress. The nurse must develop a trusting relationship with the original personality and with each of
the subpersonalities. Trust is the basis of a therapeutic relationship. Each of the personalities views itself as a
separate entity and must initially be treated as such.
100. Sleep disorders are very common in the aging individual. Sleep disturbances affect 50 percent of people age 65
and older who live at home and 66 percent of those who live in long-term care facilities .Some common causes
of sleep disturbances among elderly people include age-dependent decreases in the ability to sleep (sleep
decay); increased prevalence of sleep apnea; depression; dementia; anxiety; pain; impaired mobility;
medications; and psychosocial factors such as loneliness, inactivity, and boredom. The nurse is performing an
assessment on an older client who is having difficulty sleeping at night. Which statement, if made by the client,
indicates that teaching about improving sleep is necessary?
a. I swim three times a week.
b. I will avoid drinking alcohol at least 4 hours before going to bed
c. I drink green tea before bedtime.
d. I read for 30 minutes before bedtime.
ANSWER: C
RATIONALE: Many nonpharmacological sleep aids can be used to influence sleep. The client should avoid
caffeinated beverages and stimulants such as tea, cola, and chocolate. 1 cup of green tea (240 ml or 8 oz) contains
approx. 20-40 mg of caffeine.
OPTION A: The client should exercise regularly, because exercise promotes sleep by burning off tension that
accumulates during the day. A 20- to 30-minute walk, swim, or bicycle ride three times a week is helpful.
OPTION B: Although alcohol may help bring on sleep, after a few hours it acts as a stimulant, increasing the number
of awakenings and generally decreasing the quality of sleep later in the night. It is therefore best to limit alcohol
consumption to one to two drinks per day, or less, and to avoid drinking 4 to 6 hours before bedtime.
OPTION D: Establish a Soothing Pre-Sleep Routine - Light reading before bed is a good way to prepare yourself for
sleep. Ease the transition from wake time to sleep time with a period of relaxing activities an hour or so before bed.
Take a bath (the rise, then fall in body temperature promotes drowsiness), read a book, watch television, or practice
relaxation exercises. Avoid stressful, stimulating activitiesdoing work, discussing emotional issues. Physically and
psychologically stressful activities can cause the body to secrete the stress hormone cortisol, which is associated
with increasing alertness. If you tend to take your problems to bed, try writing them downand then putting them
aside
Sleep Hygiene Tips:
Get regular
Sleep when sleepy
Get up & try again. If you havent been able to get to sleep after about 20 minutes or more, get up and do
something calming or boring until you feel sleepy, then return to bed and try again.
Avoid caffeine & nicotine
Avoid alcohol.
Bed is for sleeping. Try not to use your bed for anything other than sleeping and sex, so that your body comes
to associate bed with sleep. If you use bed as a place to watch TV, eat, read, work on your laptop, pay bills, and
other things, your body will not learn this connection.
No naps: This is because late-day naps decrease sleep drive. If you must nap, its better to keep it short and
before 5 p.m.
Sleep rituals.

Bathtime. Having a hot bath 1-2 hours before bedtime can be useful, as it will raise your body temperature,
causing you to feel sleepy as your body temperature drops again.
No clock-watching
Use a sleep diary
Exercise: try not to do strenuous exercise in the 4 hours before bedtime.
Eat right.
Most common sleep disorders:
Insomnia - a hard time falling or staying asleep
Sleep apnea - breathing interruptions during sleep
Restless legs syndrome - a tingling or prickly sensation in the legs
Narcolepsy - daytime "sleep attacks"
Reference: Meiner, S., & Leuckenotte, A. (2006). Gerontologic nursing (3rd ed., pp. 13-14). St. Louis: Mosby.

Das könnte Ihnen auch gefallen